INSTA SUBJECT TEST - 24 2023 (WWW - Upscmaterial.online)

You might also like

Download as pdf or txt
Download as pdf or txt
You are on page 1of 118

.

Test-24 (Subject)
( Insta Prelims Test Series 2023 ) Total Marks : 200.00

1. Consider the following statements


1. A state minister is not eligible for election for the presidential candidate
2. The security money deposited by a candidate to the Reserve Bank of India during nomination
for presidential election is forfeited if he fails to secure one-sixth of the votes polled.
3. Only the supreme court is empowered to declare a presidential election as void

Which of the statements given above is/are correct?


A. 2 and 3 only
B. 2 only
C. 1 and 2 only
D. 1 and 3 only

2. Consider the following statements


1. In the Menaka case 1978, the Supreme Court ruled that protection to individuals under article
21 includes the right against arbitrary legislative action.
2. The protection against double jeopardy provided by the constitution is unavailable for non
judicial authorities.
3. The Supreme Court, under article 21A, provided that attainment of education including
professional and technical education up to any level to secure means of decent livelihood is a
fundamental right in India.

Which of the statements given above is/are correct?


A. 1 and 2 only
B. 1 only
C. 2 and 3 only
D. 1 and 3 only

3. The 91st Amendment Act of 2003 provided that


1. A member of the Parliament disqualified under provisions of Tenth schedule of the Constitution
cannot be appointed as a Minister
2. The total number of ministers including the chief ministers in the state should be more than 12.
3. The total number of ministers along with the Prime Minister shall be at least 15% of the total
strength of the Lok Sabha
4. A member of Rajya Sabha disqualified on the ground of diffraction shall also be ineligible to be
appointed under any office of the state government.

Select the correct answer using the codes given below


A. 1, 2 and 3 only
B. 2, 3 and 4 only
C. 1 and 3 only
D. All of the above

4. Consider the following statements regarding National Foundation for Communal


Harmony

instacourses.insightsonindia.com 1
www.upscmaterial.online

Join Our Official Telegram Channel

https://telegram.me/+YwhD2COlqds3N2I1

Or Scan QR Code For Join Our Channel


.
Test-24 (Subject)
( Insta Prelims Test Series 2023 ) Total Marks : 200.00

1. It is an autonomous body under Ministry of Culture


2. It is a multi member body headed by the Prime minister of India.
3. It promotes national integration by organising activities in Association with NGOs.

Which of the statements given above is/are correct?


A. 1 and 2 only
B. 3 only
C. 1 and 2 only
D. 1, 2 and 3

5. Consider the following statements regarding constituent assembly


1. The princely states and the governors’ provinces were allotted seats in equal numbers.
2. The oldest member was elected as the permanent president of the assembly following the
British practice.
3. The constituent assembly meeting was boycotted by the Muslim League.
4. The Indian Independence Act of 1947 empowered the assembly to frame a constitution as well
as enact ordinary laws for the country.

Which of the statements given above is/are correct?


A. 3 and 4 only
B. 2 and 3 only
C. 1 and 2 only
D. 1, 2 and 3 only

6. Consider the following statements


1. No foreigner holding an office of profit under the state can accept an emolument from a foreign
state without consent of the state legislature.
2. No state can confer a military title to a foreigner without consent of the president.

Which of the statements given above is/are correct?


A. 1 only
B. 2 only
C. Both 1 and 2
D. Neither 1 nor 2

7. Consider the following statements regarding State Administrative Tribunals


1. Under article 323A, these tribunals exercise original jurisdiction regarding recruitment matters
of State Government employees.
2. All the members are appointed by the governor in consultation with state legislature.
3. The chairman of a Joint Administrative Tribunal for two States is appointed by the president.

Which of the statements given above is/are correct?


A. 1 and 2 only
B. 2 and 3 only

instacourses.insightsonindia.com 2
.
Test-24 (Subject)
( Insta Prelims Test Series 2023 ) Total Marks : 200.00

C. 1 and 3 only
D. 1, 2 and 3

8. Consider the following statements regarding Impeachment of the president


1. The impeachment charges should be signed by at least half of the members of a House where it
has originated.
2. The charges are required to be passed by majority of two-thirds of total membership of both
Houses of the Parliament.
3. No House can initiate impeachment charge without giving a prior notice to the president.

Which of the statements given above is/are correct?


A. 1 and 3 only
B. 2 and 3 only
C. 1 and 2 only
D. 1, 2 and 3

9. Consider the following statements regarding Writs issued by the Supreme Court and
High courts
1. No Court can refuse to exercise it’s writ jurisdiction.
2. Unlike the Supreme Court, High courts can issue writs for enforcement of both fundamental
rights and ordinary legal rights.
3. The 15th Constitutional Amendment Act of 1963 provided that a high court can issue writs
against an authority located in any part of India.

Which of the statements given above is/are correct?


A. 2 only
B. 2 and 3 only
C. 1 only
D. 1 and 3 only

10. Consider the following statements regarding Amendment of the Constitution


1. A bill for the purpose of amendment of Fifth and Sixth schedule of the constitution is required to
be passed by a simple majority of the Parliament.
2. A bill regarding amendment of federal provisions of the constitution should be ratified by
legislatures of half of the States by simple majority.

Which of the statements given above is/are correct?


A. 1 only
B. 2 only
C. Both 1 and 2
D. Neither 1 nor 2

11. Regarding Solicitor General of India, which of the statements is incorrect?

instacourses.insightsonindia.com 3
Download From - https://upscmaterial.online/

.
Test-24 (Subject)
( Insta Prelims Test Series 2023 ) Total Marks : 200.00

A. It is a statutory office and can be abolished by an act of the Parliament


B. He is the second law officer in the country working under the Attorney General of India.
C. He is appointed by the President on recommendation of the Chief Justice of India.
D. He can appear in both Supreme Court or any High Court on behalf of the Government of
India.

12. Consider the following statements regarding Advocate General of the state
1. He enjoys all the privileges available to a member of the state legislature.
2. He is remuneration is determined by the governor of the state concerned.
3. He resigns when the council of minister resigns from office in a state.

Which of the statements given above is/are correct?


A. 2 and 3 only
B. 1 and 3 only
C. 1 only
D. 1, 2 and 3

13. Consider the following statements


1. A presidential ordinance becomes inoperative if the Parliament, after it’s reassembly, does not
approve it within six weeks.
2. The governor can promulgate or withdraw an ordinance only on advise of state Council of
ministers.

Which of the statements given above is/are correct?


A. 1 only
B. 2 only
C. Both 1 and 2
D. Neither 1 nor 2

14. Consider the following statements regarding the Constitution of India


1. The constitution under article 27 provides that no fee can be levied by the State for control of
administration of a religious institution.
2. The origin of Directive Principles of State Policy can be traced to the 'Instrument of Instructions'
enumerated in the Government of India Act 1935.

Which of the statements given above is/are correct?


A. 1 only
B. 2 only
C. Both 1 and 2
D. Neither 1 nor 2

15. Consider the following statements


1. The Chief Commissioner’s Provinces under the British were constituted as union territories

instacourses.insightsonindia.com 4

https://upscmaterial.online/
Download From - https://upscmaterial.online/

.
Test-24 (Subject)
( Insta Prelims Test Series 2023 ) Total Marks : 200.00

under provisions of First Constitutional Amendment Act 1951.


2. The president has to revoke a proclamation of National emergency when a resolution passed by
a special majority of the Lok Sabha is presented to him for the same.

Which of the statements given above is/are correct?


A. 1 only
B. 2 only
C. Both 1 and 2
D. Neither 1 nor 2

16. Consider the following statements


1. A Tribunal for the purpose of adjudication of disputes related to election to Parliament and state
legislature can be established both by Parliament and state legislatures.
2. A Bill passed by the state legislature endangering position of the state High Court should be
reserved for consideration of the president by the governor.

Which of the statements given above is/are correct?


A. 1 only
B. 2 only
C. Both 1 and 2
D. Neither 1 nor 2

17. Consider the following statements regarding the Governor


1. Appointment of an individual as a governor of two States is a parliamentary convention and not
mentioned in the Constitution.
2. The Constitution provides that the chief minister of the state concerned should be consulted
while appointment of a governor is made by the president.
3. The allowances of the governor are determined by the parliament.

Which of the statements given above is/are correct?


A. 2 and 3 only
B. 1 only
C. 3 only
D. 1 and 2 only

18. Consider the following statements


1. Certiorari can be rissued against a Tribunal for the purpose of transfer of a case pending with it
to a higher court.
2. Quo- Warranto can be issued only in case of a Constitutional authority.
3. Prohibition can be issued only against judicial and quasi-judicial authorities.

Which of the statements given above is/are correct?


A. 1 only
B. 2 and 3 only

instacourses.insightsonindia.com 5

https://upscmaterial.online/
Download From - https://upscmaterial.online/

.
Test-24 (Subject)
( Insta Prelims Test Series 2023 ) Total Marks : 200.00

C. 2 only
D. 1 and 3 only

19. Consider the following statements regarding the Delimitation Commission


1. It is appointed by the Election Commission of India for the purpose of redrawing boundaries of
Lok Sabha and state assembly seats.
2. The members are drawn from state election commissions and is chaired by the Chief Election
commissioner.
3. No order of the commission can be subjected to judicial review.

Which of the statements given above is/are correct?


A. 1 only
B. 3 only
C. 2 and 3 only
D. 1, 2 and 3

20. Consider the following statements regarding the parliament


1. A Secret sitting of the Lok Sabha can be convened only on permission of the president.
2. The fundamental right to freedom of press regarding publication of true reports of
parliamentary procedents without permission of the Lok Sabha is not applicable for a secret
sitting.

Which of the statements given above is/ are correct?


A. 1 only
B. 2 only
C. Both 1 and 2
D. Neither 1 nor 2

21. Regarding Estimates Committee, which of the following statements is incorrect?

A. All members of the committee are from the Lok Sabha, whose duty is to examine
estimates included in the budget and suggest economies in public expenditure.
B. The members are elected in a way such that all parties get due representation in the
committee.
C. The Prime minister act as chairman of the committee.
D. The origin of the committee can be traced to the pre-independence period.

22. Regarding Comptroller and Auditor General of India, which of the following
statements is correct?

A. He holds office till pleasure of the President who also determines his service conditions.
B. He can be removed on the basis of a resolution passed by both houses of the Parliament
with special majority.
C. The administrative expenses of the office of CAG forms part of expenditure made from
instacourses.insightsonindia.com 6

https://upscmaterial.online/
Download From - https://upscmaterial.online/

.
Test-24 (Subject)
( Insta Prelims Test Series 2023 ) Total Marks : 200.00

the consolidated fund of India and is subjected to voting of the Parliament.


D. He is eligible for reappointment after retirement as an auditor in public corporations only.

23. Which of the following ensures independence of the office of the speaker of Lok
Sabha?
1. His rank in the Table of Precedence is same as that of the Chief Justice of India
2. He can decide the question of disqualification of a member of parliament
3. His salaries and allowances are charged on the Consolidated Fund of India
4. He is the final interpreter of the provisions of the Constitution of India within the Lok Sabha
5. He can exercise only a casting vote in case of a tie.
6. He can be removed by the Lok Sabha only by an absolute majority.

Select the correct answer using the codes given below


A. 1, 2, 4 and 5 only
B. 1, 3, 5 and 6 only
C. 2, 4, 5 and 6 only
D. All of the above

24. Consider the following statements


1. A Remote Voting Machine is an electronic voting machine enabled to handle multiple
constituences from a single remote polling booth.
2. Electoral rolls for an Assembly constituency is prepared by the Chief Electoral Officer of a state.

Which of the statements given above is/are correct?


A. 1 only
B. 2 only
C. Both 1 and 2
D. Neither 1 nor 2

25. Consider the following statements


1. Article 394A provided for publication of an authoritative text of the Constitution of India in Hindi
language in the Gazzette of India.
2. The 58th Constitutional Amendment Act of 1987 provided that it is not mandatory to provide a
Hindi translation of every amendment of the Constitution.

Which of the statements given above is/are correct?


A. 1 only
B. 2 only
C. Both 1 and 2
D. Neither 1 nor 2

26. Consider the following statements regarding Public Accounts Committee (PAC)
1. It was initially established as a standing financial committee in post independence period on
recommendation of Krishna Menon committee.
instacourses.insightsonindia.com 7

https://upscmaterial.online/
Download From - https://upscmaterial.online/

.
Test-24 (Subject)
( Insta Prelims Test Series 2023 ) Total Marks : 200.00

2. All reports audited by the Comptroller and Auditor General of India related to appropriation
accounts are scrutinised by the committee.
3. Although the Comptroller and Auditor General is not a member of the committee, he is to assist
the committee in exercise of examining of reports submitted by him.

Which of the statements given above is/are correct?


A. 1 and 3 only
B. 1 and 2 only
C. 2 only
D. 2 and 3 only

27. Consider the following statements regarding Municipal Corporation


1. The Municipal Council consists of both directly elected and nominated members.
2. Decisions related to public works and their implementation are carried on by standing
committees of the corporation.
3. The chief executive authority of the corporation is vested in the Mayor who is elected for a
period of one year.

Which of the statements given above is/are correct?


A. 1 and 2 only
B. 1 and 3 only
C. 2 only
D. 2 and 3 only

28. Consider the following statements regarding Panel of chairpersons of Lok Sabha
1. They are nominated by the speaker from amongst the members of the House.
2. Anyone of them is to preside over the House when the office of Speaker or Deputy Speaker falls
vacant.
3. A member of this panel is selected, and appointed as the Speaker Pro Tem by the president.

Which of the statements given above is/are correct?


A. 2 only
B. 1 only
C. 2 and 3 only
D. 1 and 2 only

29. Consider the following statements


1. A Provision related to elections to the state legislature can be made by the Parliament only.
2. An election petition challenging an election to the state legislature can be tried by the supreme
court only.

Which of the statements given above is/are correct?


A. 1 only
B. 2 only

instacourses.insightsonindia.com 8

https://upscmaterial.online/
Download From - https://upscmaterial.online/

.
Test-24 (Subject)
( Insta Prelims Test Series 2023 ) Total Marks : 200.00

C. Both 1 and 2
D. Neither 1 nor 2

30. Consider the following statements


1. By ‘Adjournment’, the presiding officer of a house suspends work in a sitting of the House for a
specified time.
2. The power of ' Adjournment sine die' and ‘Prorogation’ lies with the president of India.
3. Both ‘Adjournment’ and 'Prorogation' has no effect on bills pending before a House of the
Parliament.

Which of the statements given above his correct?


A. 1 and 3 only
B. 2 and 3 only
C. 1 and 2 only
D. 1 only

31. Regarding Cabinet Committees, which of the following statements is incorrect?

A. They are extra-Constitutional bodies set up under the Rules of Business


B. Both cabinet and non cabinet ministers can be members of these committees.
C. They can either be permanent or temporary in nature.
D. All cabinet committees are set up and headed by the Speaker of the Lok Sabha.

32. Consider the following statements regarding Autonomous Districts


1. The constitution empowers the governor to organised and define boundaries of such districts.
2. An autonomous district is created for a particular tribal group.
3. The constitution empowers the President to constitute a Commission to examine administrative
system in autonomous districts.

Which of the statements given above is/are correct?


A. 1 and 3 only
B. 2 only
C. 1 only
D. 1 and 2 only

33. Consider the following statements


1. No authority can question a decision of the president regarding age of a High Court judge.
2. An individual who has served a judicial office or has been an advocate of a High Court, both for
more than five years, is eligible to be appointed as a high court judge.

Which of the statements given above is/are correct?


A. 1 only
B. 2 only

instacourses.insightsonindia.com 9

https://upscmaterial.online/
Download From - https://upscmaterial.online/

.
Test-24 (Subject)
( Insta Prelims Test Series 2023 ) Total Marks : 200.00

C. Both 1 and 2
D. Neither 1 nor 2

34. Which of the following has replaced the British Privy Council in the post-
independence period?

A. Reserve Bank of India


B. Supreme Court of India
C. Office of President
D. Council of ministers

35. Consider the following statements regarding Municipalities


1. The state legislature is authorised to provide for representation of members of Lok Sabha
representing constituencies that comprise wholly or partly the municipal area
2. The territorial area as well as composition of a ward committee are decided by the Governor.
3. The constitution provides that all persons qualified to be chosen to the state legislative Council
are qualified for being a member of municipality.

Which of the statements given above is/are correct?


A. 2 only
B. 1 and 3 only
C. 1 only
D. 1, 2 and 3

36. Consider the following statements regarding the Basic Exchange and Cooperation
Agreement (BECA):
1. It allows India to get real-time access to American geospatial intelligence enhancing the
accuracy of armed drones.
2. It allows the militaries of the United States and India to replenish from each other’s bases.
3. It allows the United States to provide India with encrypted communications systems during
times of both peace and war.

Which of the statements given above is/are correct?


A. 1 only
B. 1 and 2 only
C. 2 and 3 only
D. 1, 2 and 3

37. The Oslo Peace Accord sometimes mentioned in news was originally signed between-

A. Greece and Turkey


B. Ukraine and Russia
C. Armenia and Azerbaijan
D. Palestine and Israel

instacourses.insightsonindia.com 10

https://upscmaterial.online/
Download From - https://upscmaterial.online/

.
Test-24 (Subject)
( Insta Prelims Test Series 2023 ) Total Marks : 200.00

38. The key targets to be achieved under Immunization Agenda 2030 are-
1. Halving the number of children completely missing out on vaccines.
2. Achieving 90% coverage for essential vaccines given in adolescence.
3. Achieving full immunization coverage for all pregnant women.

Which of the statements given above is/are correct?


A. 1 only
B. 1 and 2 only
C. 2 and 3 only
D. 1, 2 and 3

39. Consider the following statements:


The Kalapani area is the largest territorial dispute between Nepal and India.
1. It is located in the easternmost corner of Uttarakhand’s Pithoragarh district.
2. It shares a border on the north with the Tibet Autonomous Region of China and Nepal in the
east.

Which of the statements given above is/are correct?


A. 1 only
B. 2 only
C. Both 1 and 2
D. Neither 1 nor 2

40. The Minsk Group was created by the Organisation for Security and Cooperation in
Europe in the context of-

A. Daraa Insurgency
B. Nagorno-Karabakh Conflict
C. Nigerian bandit Conflict
D. Conflict of Rakhine State

41. With reference to the Blue Dot Network, consider the following statements:
1. It is a part of the Act East Policy of India.
2. It aims to counter the Chinese Belt and Road Initiative.
3. It was jointly launched by Japan, India and Australia.

Which of the statements given above is/are not correct?


A. 1 only
B. 2 and 3 only
C. 1 and 3 only
D. 1, 2 and 3

42. The Copernicus Programme sometimes talked about in news is associated with-

instacourses.insightsonindia.com 11

https://upscmaterial.online/
Download From - https://upscmaterial.online/

.
Test-24 (Subject)
( Insta Prelims Test Series 2023 ) Total Marks : 200.00

A. Human exploration of the Moon


B. Earth observation programme
C. Exploration of Mars’ atmosphere
D. Exploration of Jupiter and its composition

43. Consider the following statements regarding the Wassenaar Arrangement (WA):
1. The countries in this arrangement subscribe to arms export controls similar to the Nuclear
Suppliers Group.
2. All the permanent members of the United Nations Security Council are signatories of the WA.

Which of the statements given above is/are correct?


A. 1 only
B. 2 only
C. Both 1 and 2
D. Neither 1 nor 2

44. The Ashgabat Agreement sometimes mentioned in news is a/an-

A. Agreement primarily focussing on Yemen’s humanitarian conditions.


B. Agreement on Economic Cooperation between India and the member states of the
Cooperation Council for the Arab States.
C. Agreement creating integrated multilateral trading system encompassing the General
Agreement on Tariffs and Trade.
D. Agreement that envisages the facilitation of transit and transportation of goods between
Central Asia and the Persian Gulf.

45. The Rwandan Genocide recently talked about was a consequence of-

A. The troubled relationship between majority Hutus and minority Tutsis.


B. The conflict in Darfur that was a part of an ethnic cleansing campaign.
C. The migration of Nigerian Muslims from northern Nigeria to the Niger Delta.
D. The civil conflict involving the ethno-regional militia of Ethiopia’s Tigray.

46. Consider the following:


1. Exercise Garuda-VII
2. Exercise Cyclone-I
3. Exercise Mitrashakti
4. Exercise Maitree

Which of the above mentioned defence exercises was/were conducted between India and its
immediate neighbours?
A. 1 only
B. 3 only

instacourses.insightsonindia.com 12

https://upscmaterial.online/
Download From - https://upscmaterial.online/

.
Test-24 (Subject)
( Insta Prelims Test Series 2023 ) Total Marks : 200.00

C. 3 and 4 only
D. 1, 2 and 4 only

47. Operation Ganga Initiative was launched in context of-

A. Large-scale evacuations of Indian citizens from Afghanistan.


B. Sending humanitarian relief to Donetsk region of Ukraine.
C. Bringing back the Indian citizens stranded in Ukraine.
D. Relief operation in Earthquake hit Nepal by India.

48. With reference to the Indo-Pacific Economic Framework, consider the following
statements:
1. It broadly rests on supply chain resilience and decarbonisation as one of its pillars but does not
cover the anti-corruption measures.
2. Countries are free to join initiatives under any of the stipulated pillars but are expected to
adhere to all commitments once they enrol.

Which of the statements given above is/are correct?


A. 1 only
B. 2 only
C. Both 1 and 2
D. Neither 1 nor 2

49. Which one of the following Organisations releases the Global Economic Prospects
Report?

A. The World Economic Forum


B. The International Monetary Fund
C. The World Bank Group
D. The Organisation for Economic Co-operation and Development

50. ‘The Treaty of Punakha’ significant in Indian historical context was signed between
India and-

A. Bhutan
B. Nepal
C. China
D. Sri Lanka

51. With reference to Nord Stream Pipeline, consider the following statements:
1. It is the longest subsea pipeline which runs under the Baltic Sea exporting gas to Europe.
2. The gas for Nord Steam comes from the gas condensate deposit in Western Siberia.
3. The Nord Stream crosses the Exclusive Economic Zones and well as the territorial waters of

instacourses.insightsonindia.com 13

https://upscmaterial.online/
Download From - https://upscmaterial.online/

.
Test-24 (Subject)
( Insta Prelims Test Series 2023 ) Total Marks : 200.00

both Denmark and Germany.

Which of the statements given above is/are correct?


A. 1 only
B. 1 and 2 only
C. 2 and 3 only
D. 1, 2 and 3

52. With reference to Teesta River Dispute, consider the following statements:
1. It is a distributary of the Barak River which flows through Assam and then flows on to
Bangladesh.
2. Bangladesh seeks an equitable distribution of Teesta waters on the lines of the Ganga Water
Treaty of 1996.
3. The Rahimpur Canal project was originally built to give access of Teesta’s water to Assam
during the lean season.

Which of the statements given above is/are correct?


A. 1 only
B. 2 only
C. 2 and 3 only
D. 1 and 3 only

53. With reference to the Eastern Economic Forum (EEF), consider the following
statements:
1. The primary objective of the EEF is to increase the Foreign Direct Investments in the region that
encompasses one-third of Russia’s territory.
2. India has abstained itself from investing in EEF due to the current international conditions
including Russia-Ukraine conflict and China’s involvement in EEF.

Which of the statements given above is/are correct?


A. 1 only
B. 2 only
C. Both 1 and 2
D. Neither 1 nor 2

54. With reference to Quadrilateral Strategic Dialogue (QUAD), consider the following
statements:
1. Quad is not a structured multilateral organisation and lacks a secretariat and any permanent
decision-making body.
2. Unlike creating policy on the lines of United Nations, Quad has focused on expanding existing
agreements between member countries.
3. Like the North Atlantic Treaty Organisation, Quad includes provisions for collective defence as
well as joint military exercises.

Which of the statements given above is/are correct?


instacourses.insightsonindia.com 14

https://upscmaterial.online/
Download From - https://upscmaterial.online/

.
Test-24 (Subject)
( Insta Prelims Test Series 2023 ) Total Marks : 200.00

A. 1 only
B. 1 and 2 only
C. 2 and 3 only
D. 1, 2 and 3

55. Consider the following statements regarding the International Monetary Fund:
1. It is a specialized agency of the United Nations and its membership is open to those countries
who accept its statutes.
2. It provides the Extended Fund Facility by which countries can borrow money for a short-term
balance of payments.
3. It is funded by quota subscriptions paid by member states and the size of the quota is
independent of the size of the member's economy.

Which of the statements given above is/are correct?


A. 1 only
B. 2 and 3 only
C. 1 and 2 only
D. 1, 2 and 3

56. Consider the following statements:


1. After the Brexit, Northern Ireland remained the only constituent of United Kingdom that shared
a land border with a European Union member.
2. Northern Ireland Protocol is a post-Brexit agreement that has created a trade border between
Northern Ireland and the rest of the United Kingdom.

Which of the statements given above is/are correct?


A. 1 only
B. 2 only
C. Both 1 and 2
D. Neither 1 nor 2

57. Consider the following statements:


1. The boundary between China and Bhutan has never been delimited and Bhutan has no formal
diplomatic relations with China.
2. The Sakteng Wildlife Sanctuary in Bhutan bordering Arunachal Pradesh is the oldest disputed
territory between Bhutan and China.

Which of the statements given above is/are correct?


A. 1 only
B. 2 only
C. Both 1 and 2
D. Neither 1 nor 2

instacourses.insightsonindia.com 15

https://upscmaterial.online/
Download From - https://upscmaterial.online/

.
Test-24 (Subject)
( Insta Prelims Test Series 2023 ) Total Marks : 200.00

58. The Davos Summit often mentioned in news is organised by-

A. World Cities Culture Forum


B. World Economic Forum
C. Amnesty International
D. Oxfam International

59. With reference to Carbon Border tax, consider the following statements:
1. It involves imposing an import duty on a product manufactured in a country with more lax
climate rules than the one buying it.
2. A group of countries including India has adopted the carbon border taxes policy at the COP27.
3. It has been criticised as it can result in market distortion and aggravate the trust deficit
amongst the participating countries.

Which of the statements given above is/are correct?


A. 1 only
B. 2 and 3 only
C. 1 and 3 only
D. 1, 2 and 3

60. With reference to the Black Sea Grain Initiative, consider the following statements:
1. It provided for a safe maritime humanitarian corridor for Ukrainian exports.
2. It created a Joint Coordination Centre comprising representatives from the United Nations.
3. It been credited for making a substantial difference to the global cost of living crisis.

Which of the statements given above is/are correct?


A. 1 only
B. 1 and 2 only
C. 2 and 3 only
D. 1, 2 and 3

61. The Supply Chain Resilience Initiative often heard in news was initiated by which of
the following countries?
1. Australia
2. Japan
3. Russia
4. China
5. India

Select the correct answer using the code given below:


A. 1, 2, 3 and 4 only
B. 3 and 5 only
C. 1, 2 and 5 only
D. 2, 3 and 4 only

instacourses.insightsonindia.com 16

https://upscmaterial.online/
Download From - https://upscmaterial.online/

.
Test-24 (Subject)
( Insta Prelims Test Series 2023 ) Total Marks : 200.00

62. The Gogra Post recently seen in news is located in-

A. Eastern Ladakh
B. Eastern Arunachal Pradesh
C. Chungthang Valley of Sikkim
D. Dzukou Valley of Nagaland

63. The Falklands Islands disputed between the United Kingdom and Argentina are
located in-

A. North Pacific Ocean


B. South Atlantic Ocean
C. Indian Ocean
D. Southern Ocean

64. The Sendai Framework was recently mentioned in news in the context of-

A. First-Level Referral Health Facilities


B. Development of Indo-Pacific
C. Combatting global terrorism
D. Disaster Risk Reduction

65. Consider the following countries:


1. Kenya
2. Sudan
3. Somalia
4. Tanzania

Which of the above mentioned countries comprise of the Horn of Africa?


A. 1 only
B. 3 only
C. 2 and 4 only
D. 1, 2, 3 and 4

66. Consider the following statements regarding Indian polity


1. The death of an incumbent Prime Minister automatically dissolves the Council of Ministers.
2. The Deputy ministers are not members of the cabinet and are not given independent charge of
any department.
3. The 42nd Constitutional Amendment Act of 1976 introduced the concept of ‘Cabinet’ in the
Constitution.

Which of the statements given above is/are correct?


A. 1 and 3 only

instacourses.insightsonindia.com 17

https://upscmaterial.online/
Download From - https://upscmaterial.online/

.
Test-24 (Subject)
( Insta Prelims Test Series 2023 ) Total Marks : 200.00

B. 1 and 2 only
C. 2 and 3 only
D. 3 only

67. Consider the following statements regarding the National Commission for Scheduled
Tribes
1. It is a multi-member constitutional body whose members are elected for a period of three years.
2. The conditions of service of the members are determined by the president.
3. The Constitution has empowered the commission with powers of a Civil Court in relation to
summoning any individual from any part of India and examining him on oath.

Which of the statements given above is/are correct?


A. 1 only
B. 2 and 3 only
C. 2 only
D. 1, 2 and 3

68. Consider the following statements regarding Vacation Bench


1. A Vacation Bench can be appointed by the president only
2. Only the Chief Justice of India can preside over a vacation bench.
3. Any advocate could approach the Vacation Officer of the supreme court to seek relief on urgent
matters.

Which of the statements given above is/are correct?


A. 3 only
B. 2 only
C. 1 and 2 only
D. 1 and 3 only

69. Consider the following statements


1. An individual arrested under an ordinary law should be released after 24 hours unless a
magistrate authorizes his further detention.
2. It is a constitutional right of an individual arrested under an ordinary law to be defended by a
legal practitioner.

Which of the statements above is/are correct?


A. 1 only
B. 2 only
C. Both 1 and 2
D. Neither 1 nor 2

70. Consider the following statements regarding National emergency


1. A resolution for national emergency declaration should be passed by both houses of the
Parliament by a special majority within one month of its proclamation.
instacourses.insightsonindia.com 18

https://upscmaterial.online/
Download From - https://upscmaterial.online/

.
Test-24 (Subject)
( Insta Prelims Test Series 2023 ) Total Marks : 200.00

2. During the time of National Emergency the normal tenure of both Lok Sabha and state
legislative assembly can be extended by a law of parliament by one year.

Which of the statements given above is/are correct?


A. 1 only
B. 2 only
C. Both 1 and 2
D. Neither 1 nor 2

71. Consider the following statements regarding removal of High Court judges
1. A removal motion for such purpose can be initiated in either House of the Parliament.
2. The charges against a judge should be inquired into by a committee headed by the Chief Justice
of India.
3. The removal motion should be passed by each house of the Parliament by a special majority.

Which of the statements given above is/are correct?


A. 2 only
B. 1 and 3 only
C. 2 and 3 only
D. 1 only

72. Consider the following statements regarding Financial Emergency


1. A resolution regarding financial emergency cannot be issued when the Lok Sabha has been
dissolved.
2. Once approved, the emergency continues for one financial year.
3. President can revoke financial emergency without approval of the Parliament.
4. A Direction for reduction in salaries of High Court Judges during financial emergency maybe
issued by the president only.

Which of the statements given above is/are correct?


A. 3 and 4 only
B. 1, 2 and 3 only
C. 1, 2 and 4 only
D. 2, 3 and 4 only

73. Consider the following statements

1. Money can be appropriated out of the Consolidated fund of India only in accordance with a
parliamentary law based on recommendations of Comptroller and Auditor General of India
(CAG).
2. The Contingency Fund of India is held by the finance secretary on behalf of the president.

Which of the statements given above is/are correct?


A. 1 only

instacourses.insightsonindia.com 19

https://upscmaterial.online/
Download From - https://upscmaterial.online/

.
Test-24 (Subject)
( Insta Prelims Test Series 2023 ) Total Marks : 200.00

B. 2 only
C. Both 1 and 2
D. Neither 1 nor 2

74. Consider the following statements regarding Adjournment Motion


1. Only the Lok Sabha can introduce this motion.
2. At least half of the members of the House should support this motion, which deals with a matter
of urgent public importance.
3. The discussion on such a motion should last for at least one business day.

Which of the statements given above is/are correct?


A. 1 only
B. 3 only
C. 2 and 3 only
D. 1, 2 and 3

75. Consider the following statements


1. The state should secure the right to public assistance in case of unemployment and old age
2. The state should work to raise the level of nutrition and improve public health.
3. The state should provide it consumption of intoxicating drugs that are harmful for health
4. The state should promote educational and economic interests of weaker sections of the society
and protect them from social injustice.

Which of the above are included in Socialist Principles under the Directive Principles of State
Policy?
A. 1 and 2 only
B. 2, 3 and 4 only
C. 2 and 3 only
D. 1, 3 and 4 only

76. Consider the following statements regarding the Central Vigilance Commission
1. The commission was established as a statutory body on recommendations of the Santhanam
Committee on Prevention on Corruption.
2. The members are appointed based on recommendation of a committee headed by the Prime
minister of India.
3. A member of this commission is not eligible for further employment under Central or state
government.
4. In case the central government does not agree with advice of the CVC on any Vigilance issue,
the reason shall be communicated to the commission.

Which of the statements given above is/are correct?


A. 2 and 4 only
B. 1, 3 and 4 only
C. 1, 2 and 3 only

instacourses.insightsonindia.com 20

https://upscmaterial.online/
Download From - https://upscmaterial.online/

.
Test-24 (Subject)
( Insta Prelims Test Series 2023 ) Total Marks : 200.00

D. All of the above

77. Consider the following statements


1. No religious instructions can be propagated by any institution maintained completely by State
funds.
2. The constitutional right to perform religious worship and exhibit one's beliefs related to religion
is available only to citizens of India.

Which of the statements given above is/are correct?


A. 1 only
B. 2 only
C. Both 1 and 2
D. Neither 1 nor 2

78. Consider the following statements regarding Central Information Commission


1. No member of the Parliament or state legislature can be appointed by the president as a
member of the commission.
2. The annual report of the commission is required to be laid before each house of the Parliament.
3. The commission is empowered to direct a public authority to appoint a Public Information
Officer.

Which of the statements given above is/are correct?


A. 2 only
B. 1 and 3 only
C. 1 and 2 only
D. 1,2 and 3

79. Which of the following is not included in the Fundamental Duties as enshrined in the
Constitution?

A. An individual should provide opportunities for education of his child between the age of 6
and 14 years.
B. A citizen of India should protect natural environment.
C. The state should provide for instruction in mother language at the primary stage of
education to children belonging to linguistic minority group.
D. A citizen of India should safeguard public property.

80. The consequences of proclamation of National Emergency under article 352 include
1. State legislatures are suspended and they come under complete control of the Centre
2. President can issue ordinances on the state subjects.
3. President can cancel transfer of finances from the Centre to the States.

Select the correct answer using the codes given below


A. 1 and 3 only
instacourses.insightsonindia.com 21

https://upscmaterial.online/
Download From - https://upscmaterial.online/

.
Test-24 (Subject)
( Insta Prelims Test Series 2023 ) Total Marks : 200.00

B. 2 and 3 only
C. 2 only
D. 1, 2 and 3

81. With reference to the Mission SAGAR, consider the following statements:
1. It was launched by the Indian Ocean Rim Association which is aimed at strengthening the Indian
Ocean region.
2. In Mission Sagar, India became the first responder to Mauritius and Seychelles by sending life-
saving drugs.
3. It was launched to provide Covid-19 related assistance to the island nations including Maldives
and Mauritius.

Which of the statements given above is/are correct?


A. 1 only
B. 1 and 2 only
C. 2 and 3 only
D. 1, 2 and 3

82. 1. Consider the following pairs:


Declarations in news- Adopted Objectives

1. Sirte Declaration- Peace and Security in Africa

2. Delhi Declaration Counter Terror Financing


3. Rome declaration Promote Global Tourism

How many of the pairs given above is/are correctly matched?


A. Only one pair
B. Only two pairs
C. All three pairs
D. None of the pairs

83. The Project DANTAK is sometimes talked about in context of-

A. Constructing the pioneering motorable roads in Bhutan.


B. Largest-ever infrastructure project of India in the Maldives.
C. Developing the Colombo and Trincomalee ports in Sri Lanka.
D. Building the Darchulla Bridge connecting India and Nepal.

84. Consider the following statements:


1. Senkaku islands are located in close proximity to Taiwan’s Eastern coast.
2. Senkaku Islands are associated with geopolitical tensions in the East China Sea.

instacourses.insightsonindia.com 22

https://upscmaterial.online/
Download From - https://upscmaterial.online/

.
Test-24 (Subject)
( Insta Prelims Test Series 2023 ) Total Marks : 200.00

Which of the statements given above is/are correct?


A. 1 only
B. 2 only
C. Both 1 and 2
D. Neither 1 nor 2

85. With reference to the AUKUS Partnership, consider the following statements:
1. It is a trilateral security partnership for the Indo-Pacific region.
2. It has been initiated by Australia and includes all the QUAD members.
3. It focusses on emerging quantum technologies and undersea capabilities.

Which of the statements given above is/are correct?


A. 1 only
B. 2 only
C. 1 and 3 only
D. 2 and 3 only

86. The disputed Kuril Islands sometimes talked about in news are claimed by-

A. Russia and China


B. China and Japan
C. Vietnam and Taiwan
D. Russia and Japan

87. Consider the following statements:


1. The Hague Convention is an international agreement that has been signed and ratified by India
in respect of inter-country adoptions.
2. The Montreux Convention provides international legal standards that protect the rights of all
individuals involved in migration.

Which of the statements given above is/are correct?


A. 1 only
B. 2 only
C. Both 1 and 2
D. Neither 1 nor 2

88. With reference to Farzad B Gas field, consider the following statements:
1. It is located between the Iranian and Saudi territories.
2. It was discovered by India’s ONGC Videsh Ltd (OVL) in 2000.
3. It has been recently handed over to India by Iran for Gas exploration.

Which of the statements given above is/are correct?


A. 1 only

instacourses.insightsonindia.com 23

https://upscmaterial.online/
Download From - https://upscmaterial.online/

.
Test-24 (Subject)
( Insta Prelims Test Series 2023 ) Total Marks : 200.00

B. 1 and 2 only
C. 2 and 3 only
D. 1, 2 and 3

89. The Houthis are associated with an Islamist revivalist movement primarily based in-

A. Jordan
B. Libya
C. Yemen
D. Sudan

90. Consider the following statements:


1. The Sinai Peninsula is a desert region situated between the Red Sea and the Mediterranean Sea
2. The Sinai Peninsula is bordered to the east by the Gaza Strip and the Suez Canal to the west.
3. The Egyptian army launched an operation in Sinai to get rid of aggressive branch of the Islamic
State.

Which of the statements given above is/are correct?


A. 1 and 2 only
B. 2 and 3 only
C. 1 and 2 only
D. 1, 2 and 3

91. The Global Biofuel Alliance (GIBA) is a prioritised Initiative under-

A. National Hydrogen Energy Mission


B. India’s G20 Presidency
C. National Policy on Biofuels
D. Association of Southeast Asian Nations

92. Consider the following statements:


1. The Indian Constitution does not allow the aldermen of a municipality the right to vote in its
meetings.
2. As per the Delhi Municipal Corporation Act of 1957, the Mayor, or in his absence the Deputy
Mayor has to preside over every meeting of the corporation.

Which of the statements given above is/are correct?


A. 1 only
B. 2 only
C. Both 1 and 2
D. Neither 1 nor 2

93. Consider the following statements:

instacourses.insightsonindia.com 24

https://upscmaterial.online/
Download From - https://upscmaterial.online/

.
Test-24 (Subject)
( Insta Prelims Test Series 2023 ) Total Marks : 200.00

1. In India, the practice of the President addressing Parliament can be found only in the
Government of India Act of 1935.
2. The Indian Constitution gives the President the power to address only a joint sitting of the two
Houses of Parliament.
3. The speech that the President reads in the Motion of Thanks is the viewpoint of the government
and is written by it.

Which of the statements given above is/are not correct?


A. 3 only
B. 2 and 3 only
C. 1 and 2 only
D. 1, 2 and 3

94. The species ‘Dickinsonia’ recently has been mentioned in news in context of-

A. The Earth’s oldest animal


B. The Earth’s oldest cryptogam
C. World’s oldest vertebrate found in Egypt
D. Fossil belonging to the family of Protecetids

95. Consider the following statements:


1. The Atal Innovation Mission is India’s flagship initiative to promote a culture of innovation and
entrepreneurship.
2. Urban-20 is one of the Engagement Groups under the Atal Innovation Mission to facilitate urban
development in India.

Which of the statements given above is/are correct?


A. 1 only
B. 2 only
C. Both 1 and 2
D. Neither 1 nor 2

96. The Reserve Bank of India has recently issued an Alert list to-

A. Recognise the non-performing assets of the banks.


B. Restrict the rising inflation running above its targets.
C. Create awareness against Chit Fund Business in India.
D. Caution against undertaking unauthorised forex transactions.

97. With reference to the Market Access Initiative Scheme, consider the following
statements:
1. It is strictly an Export Promotion Scheme that acts as a catalyst to promote India’s exports.
2. As a drawback, the level of assistance for each eligible activity has not been fixed under the
Scheme.
instacourses.insightsonindia.com 25

https://upscmaterial.online/
Download From - https://upscmaterial.online/

.
Test-24 (Subject)
( Insta Prelims Test Series 2023 ) Total Marks : 200.00

3. Only the Apex Trade Bodies recognized under Foreign Trade Policy of India are eligible agencies
under the initiative.

Which of the statements given above is/are correct?


A. 1 only
B. 1 and 2 only
C. 2 and 3 only
D. 1, 2 and 3

98. Which one of the following India-US Defence Exercise includes combatting bio terror
attacks?

A. Exercise Malabar
B. Exercise Tarkash
C. Exercise Yudh Abhyas
D. Exercise Sampriti

99. With reference to the SWAYAM Programme, consider the following statements:
1. It is aimed at providing encouragement and support to especially abled children to pursue
Technical Education.
2. It is a Ministry of Human Resource Development Scheme implemented by the All India Council
for Technical Education (AICTE).
3. It conducts on-line certification courses for which exams are held every semester in a hybrid
mode.

Which of the statements given above is/are not correct?


A. 2 only
B. 3 only
C. 2 and 3 only
D. 1 and 3 only

100. The Yuva Sangam Portal has been recently launched to-

A. Build indigenous technology for deployment of renewable energy.


B. Build discipline & skill youth with military ethos in civil society.
C. Build empathy and solidarity with the Indian Soldiers and the martyrs.
D. Build empathy between youth of the North Eastern States and other States.

instacourses.insightsonindia.com 26

https://upscmaterial.online/
Download From - https://upscmaterial.online/

.
Total Marks : 200.00
Test-24 (Subject)
( Insta Prelims Test Series 2023 )

1. Consider the following statements


1. A state minister is not eligible for election for the presidential candidate
2. The security money deposited by a candidate to the Reserve Bank of India during nomination
for presidential election is forfeited if he fails to secure one-sixth of the votes polled.
3. Only the supreme court is empowered to declare a presidential election as void

Which of the statements given above is/are correct?


A. 2 and 3 only
B. 2 only
C. 1 and 2 only
D. 1 and 3 only

Correct Answer : A

Answer Justification :

A person to be eligible for election as president should not hold any office of profit under
the union or any state government

However, a sitting President or Vice President of the union, Governor of state


and a minister of the union or any state is not deemed to hold any office of
profit and hence qualified as a presidential candidate.

Hence statement 1 is incorrect.

The nomination of a candidate for election to the office of the President must be
subscribed by at least 50 electors as proposers and 50 electors as secondars.

Every candidate has to make a security deposit of Rs. 15000 in the Reserve Bank of India

The security deposit is liable to be forfeited in case the candidate fails to secure
one sixth of the votes polled.

Hence statement 2 is correct.

All doubts and disputes in connection with the election of the President are inquired into
and decided by the supreme court whose decision is final

the Supreme Court can declare such an election as void.

instacourses.insightsonindia.com 1

https://upscmaterial.online/
Download From - https://upscmaterial.online/

.
Total Marks : 200.00
Test-24 (Subject)
( Insta Prelims Test Series 2023 )

Hence statement 3 is correct.

2. Consider the following statements


1. In the Menaka case 1978, the Supreme Court ruled that protection to individuals under article
21 includes the right against arbitrary legislative action.
2. The protection against double jeopardy provided by the constitution is unavailable for non
judicial authorities.
3. The Supreme Court, under article 21A, provided that attainment of education including
professional and technical education up to any level to secure means of decent livelihood is a
fundamental right in India.

Which of the statements given above is/are correct?


A. 1 and 2 only
B. 1 only
C. 2 and 3 only
D. 1 and 3 only

Correct Answer : A

Answer Justification :

In the Menaka case 1978, the supreme court provided for a wider interpretation of article
21 (Protection of life and personal liberty)

It ruled that the right to life and personal liberty of a person can be deprived by a law
provided the procedure prescribed by that law is reasonable

Hence the protection against this article should be available not only against
arbitrary executive action but also against arbitrary legislative action

Hence statement 1 is correct.

Article 20 of the constitution provides that no person shall be prosecuted and punished
for the same offence more than once.

This protection against double jeopardy is available only in case of proceedings


before a court of law or a judicial Tribunal.

It is not available in proceedings before departmental or administrative


authorities as they are not of judicial nature.

instacourses.insightsonindia.com 2

https://upscmaterial.online/
Download From - https://upscmaterial.online/

.
Total Marks : 200.00
Test-24 (Subject)
( Insta Prelims Test Series 2023 )

Hence statement 2 is correct.

Article 21 A declares that the state shall provide free and compulsory education
to all children in the age group of 6 to 14 years in such a manner as the state
may determine

This provision makes only elementary education as a fundamental right and not
higher or professional education

th
This provision was added by the 86 Constitutional Amendment Act of 2002

In 1993 the Supreme Court recognised the right to primary education as a fundamental
right to life under article 21.

Hence statement 3 is incorrect.

3. The 91st Amendment Act of 2003 provided that


1. A member of the Parliament disqualified under provisions of Tenth schedule of the Constitution
cannot be appointed as a Minister
2. The total number of ministers including the chief ministers in the state should be more than 12.
3. The total number of ministers along with the Prime Minister shall be at least 15% of the total
strength of the Lok Sabha
4. A member of Rajya Sabha disqualified on the ground of diffraction shall also be ineligible to be
appointed under any office of the state government.

Select the correct answer using the codes given below


A. 1, 2 and 3 only
B. 2, 3 and 4 only
C. 1 and 3 only
D. All of the above

Correct Answer : D

Answer Justification :

The 91st Amendment Act 2003 provided that a member of either House of Parliament
belonging to any political party, if gets disqualified on the grounds of defection, shall also
be disqualified to be appointed as a minister.

Hence statement 1 is correct.

instacourses.insightsonindia.com 3

https://upscmaterial.online/
Download From - https://upscmaterial.online/

.
Total Marks : 200.00
Test-24 (Subject)
( Insta Prelims Test Series 2023 )

The total number of ministers including the chief minister in the Council of ministers in a
state shall not exceed 15% of the total strength of the legislative assembly of that state

But the number of ministers including the Chief minister shall not be less than
12; this provision was added by the 91st Amendment Act of 2003.

Hence statement 2 is correct.

According to article 75 of the constitution, the Prime Minister along with other ministers
shall not exceed 15% of the total strength of the Lok Sabha.

Hence statement 3 is correct

The act provides that a member of either House of the Parliament or either House of a
state legislature belonging to any political party who is disqualified on the ground of
defection shall also be disqualified to hold any remunerative political post

It includes any office under the central government or state government where the
remuneration for such office is paid out of public revenue of the concerned government.

Hence statement 4 is correct.

4. Consider the following statements regarding National Foundation for Communal


Harmony
1. It is an autonomous body under Ministry of Culture
2. It is a multi member body headed by the Prime minister of India.
3. It promotes national integration by organising activities in Association with NGOs.

Which of the statements given above is/are correct?


A. 1 and 2 only
B. 3 only
C. 1 and 2 only
D. 1, 2 and 3

Correct Answer : B

Answer Justification :

The National Foundation for Communal Harmony was set up as an autonomous body in
1992

instacourses.insightsonindia.com 4

https://upscmaterial.online/
Download From - https://upscmaterial.online/

.
Total Marks : 200.00
Test-24 (Subject)
( Insta Prelims Test Series 2023 )

It works under administrative control of Union Home Ministry

It aims to promote communal harmony, fraternity and national integration.

Hence statement 1 is incorrect.

It is a multi-member body consists of official and non-official members

The official members include the Union Home minister, Minister of Education, Minister of
Information and broadcasting, Minister of Social justice and Empowerment among others.

It is chaired by Union Home minister.

Hence statement 2 is incorrect.

It provides financial assistance to the child victims social violence for their Care,
Education and training.

It promotes communal harmony and national integration by organising variety of


activities either independently or in association with educational institutions, NGOs and
other organisations.

It also conducts studies and grant scholarships to institutions and scholars for conducting
studies

It also involves Central and state governments, commercial organisations, NGOs, in


promoting objectives of the foundation.

Hence statement 3 is correct.

5. Consider the following statements regarding constituent assembly


1. The princely states and the governors’ provinces were allotted seats in equal numbers.
2. The oldest member was elected as the permanent president of the assembly following the
British practice.
3. The constituent assembly meeting was boycotted by the Muslim League.
4. The Indian Independence Act of 1947 empowered the assembly to frame a constitution as well
as enact ordinary laws for the country.

instacourses.insightsonindia.com 5

https://upscmaterial.online/
Download From - https://upscmaterial.online/

.
Total Marks : 200.00
Test-24 (Subject)
( Insta Prelims Test Series 2023 )

Which of the statements given above is/are correct?


A. 3 and 4 only
B. 2 and 3 only
C. 1 and 2 only
D. 1, 2 and 3 only

Correct Answer : A

Answer Justification :

The constituent assembly was constituted in November 1946 under the scheme
formulated by the Cabinet Mission Plan.

The total strength of it was to be 389; out of which 296 seats were allotted to the British
India and 93 States to the princely states

out of the 296 seats allotted to the British India 292 members were to be drawn from the
11 governors' provinces and four from four chief Commissioner’s provinces drawing one
from each.

Hence statement 1 is incorrect.

The constituent assembly held its first meeting on December 9, 1946

The meeting was attended by 211 members

Dr Sachhidanand Sinha, the oldest member was elected as temporary president


of the assembly following the French practice

Later on, in December 11, 1946, Dr Rajendra Prasad and HC Mukherjee were elected as
the President and Vice President of the assembly respectively.

Hence statement 2 is incorrect.

The Muslim League boycotted the first meeting of the constituent assembly held
in December 1946

They insisted on formation of a separate state of Pakistan.

instacourses.insightsonindia.com 6

https://upscmaterial.online/
Download From - https://upscmaterial.online/

.
Total Marks : 200.00
Test-24 (Subject)
( Insta Prelims Test Series 2023 )

Hence statement 3 is correct.

The India Independence Act 1947 provided that

The assembly was to be a fully sovereign body which could frame any constitution it
wanted

It also became a legislative body

in other words, two separate functions were assigned to the assembly viz., making of
constitution for free India and enacting ordinary laws for the country.

When the assembly met as a constituent body,it was chaired by Dr Rajendra Prasad and
when it met as a legislative body, it was chaired by G. V. Mavlankar.

Hence statement 4 is correct.

6. Consider the following statements


1. No foreigner holding an office of profit under the state can accept an emolument from a foreign
state without consent of the state legislature.
2. No state can confer a military title to a foreigner without consent of the president.

Which of the statements given above is/are correct?


A. 1 only
B. 2 only
C. Both 1 and 2
D. Neither 1 nor 2

Correct Answer : D

Answer Justification :

Article 18 of the constitution provides for evolution of titles and makes certain provisions
in this regard.

Under it no foreigner or citizen holding any office of profit or trust under the state can
accept any present emolument or office from or under any foreign state without consent
of the president.

Hence statement 1 is incorrect.

instacourses.insightsonindia.com 7

https://upscmaterial.online/
Download From - https://upscmaterial.online/

.
Total Marks : 200.00
Test-24 (Subject)
( Insta Prelims Test Series 2023 )

Article 18 of the constitution prohibits a state from conferring any title (except a military
or academic distinction) on any body, whether a citizen or a foreigner.

Also, a citizen of India is prohibited from accepting any title from any foreign state.

Hence statement 2 is incorrect.

7. Consider the following statements regarding State Administrative Tribunals


1. Under article 323A, these tribunals exercise original jurisdiction regarding recruitment matters
of State Government employees.
2. All the members are appointed by the governor in consultation with state legislature.
3. The chairman of a Joint Administrative Tribunal for two States is appointed by the president.

Which of the statements given above is/are correct?


A. 1 and 2 only
B. 2 and 3 only
C. 1 and 3 only
D. 1, 2 and 3

Correct Answer : C

Answer Justification :

In undertaking article 323A, the Parliament passed the Administrative Tribunals act in
1985.

This act empowers the central government to establish State Administrative Tribunals on
specific request of concerned state governments

These tribunals exercise original jurisdiction in relation to recruitment and all


service matters of State Government employees.

Hence statement 1 is correct.

The Chairman and members of State Administrative tribunals are appointed by the
president after consultation with the governor of the state concerned.

Hence statement 2 is incorrect.

instacourses.insightsonindia.com 8

https://upscmaterial.online/
Download From - https://upscmaterial.online/

.
Total Marks : 200.00
Test-24 (Subject)
( Insta Prelims Test Series 2023 )

The act also provides for setting up of Joint Administrative Tribunal for two or more States.

A JAT exercises all the jurisdiction and powers exercised by administrative tribunals for
such States

The Chairman and members of a JAT are appointed by the president after
consultation with the governors of the concerned States.

Hence statement 3 is correct.

8. Consider the following statements regarding Impeachment of the president


1. The impeachment charges should be signed by at least half of the members of a House where it
has originated.
2. The charges are required to be passed by majority of two-thirds of total membership of both
Houses of the Parliament.
3. No House can initiate impeachment charge without giving a prior notice to the president.

Which of the statements given above is/are correct?


A. 1 and 3 only
B. 2 and 3 only
C. 1 and 2 only
D. 1, 2 and 3

Correct Answer : B

Answer Justification :

The president can be removed from office by the process of impeachment for violation of
the Constitution

The impeachment charges can be initiated by either House of the Parliament.

These charges should be signed by one-fourth of the members of the house that
frames the charges.

Hence statement 1 is incorrect.

After the impeachment resolution is passed by a majority of two thirds of total


membership of the house that frames the charges, it is sent to the other house

instacourses.insightsonindia.com 9

https://upscmaterial.online/
Download From - https://upscmaterial.online/

.
Total Marks : 200.00
Test-24 (Subject)
( Insta Prelims Test Series 2023 )

The other house should investigate the charges and if it sustains the charges
and passes the impeachment resolution by a majority of two thirds of the total
membership, the president stands removed from his office.

Hence statement 2 is correct.

A 14 days’ notice should be given to the President while framing impeachment


charges against him.

Hence statement 3 is correct.

9. Consider the following statements regarding Writs issued by the Supreme Court and
High courts
1. No Court can refuse to exercise it’s writ jurisdiction.
2. Unlike the Supreme Court, High courts can issue writs for enforcement of both fundamental
rights and ordinary legal rights.
3. The 15th Constitutional Amendment Act of 1963 provided that a high court can issue writs
against an authority located in any part of India.

Which of the statements given above is/are correct?


A. 2 only
B. 2 and 3 only
C. 1 only
D. 1 and 3 only

Correct Answer : A

Answer Justification :

A remedy under article 32 (writ jurisdiction of Supreme Court) is itself a fundamental right
and hence the apex court may not refuse to exercise such a jurisdiction

On the other hand, a remedy under article 226 (Writ jurisdiction of High Court) is
discretionary and hence a High Court may refuse to exercise such power.

Hence statement 1 is incorrect.

The supreme court can issue writs only for enforcement of fundamental rights

But High Courts can issue writs for any other purpose including enforcement of an

instacourses.insightsonindia.com 10

https://upscmaterial.online/
Download From - https://upscmaterial.online/

.
Total Marks : 200.00
Test-24 (Subject)
( Insta Prelims Test Series 2023 )

ordinary legal right.

Hence statement 2 is correct.

The supreme court can issue writs against a person or government throughout the
territory of India.

But a high court can issue writs against a person residing or against the government or
authority located within its territorial jurisdiction only

th
The 15 Constitutional Amendment Act of 1963 provided that the writ
jurisdiction of High Court extends to a person or authority located outside its
territorial jurisdiction only if the cause of action arises within the territorial
jurisdiction of the concerned High Court.

Hence statement 3 is incorrect.

10. Consider the following statements regarding Amendment of the Constitution


1. A bill for the purpose of amendment of Fifth and Sixth schedule of the constitution is required to
be passed by a simple majority of the Parliament.
2. A bill regarding amendment of federal provisions of the constitution should be ratified by
legislatures of half of the States by simple majority.

Which of the statements given above is/are correct?


A. 1 only
B. 2 only
C. Both 1 and 2
D. Neither 1 nor 2

Correct Answer : C

Answer Justification :

A number of provisions in the constitution can be amended by a simple majority of two


houses of the Parliament outside the scope of 368.

The fifth schedule (dealing with administration of Scheduled areas and


Scheduled Tribes) and the sixth schedule (dealing with administration of tribal
areas) can be amended by simple majority of the Parliament.

instacourses.insightsonindia.com 11

https://upscmaterial.online/
Download From - https://upscmaterial.online/

.
Total Marks : 200.00
Test-24 (Subject)
( Insta Prelims Test Series 2023 )

Along with these, certain other provisions including the second Schedule, use of official
language, citizenship, elections to the Parliament and state legislature, formation of new
States, salaries and allowances of members of parliament, privileges of the Parliament,
delimitation of constituencies, union territories, etc., can also be amended by a simple
majority.

Hence statement 1 is correct.

The procedure for amendment of the Constitution has been provided in Article 368.

If an Amendment Bill aims to amend federal provisions of the Constitution, it is required


to be ratified by legislatures of half of the States by a simple majority that is a majority of
the members of the house present and voting.

Hence statement 2 is correct.

11. Regarding Solicitor General of India, which of the statements is incorrect?

A. It is a statutory office and can be abolished by an act of the Parliament


B. He is the second law officer in the country working under the Attorney General of India.
C. He is appointed by the President on recommendation of the Chief Justice of India.
D. He can appear in both Supreme Court or any High Court on behalf of the Government of
India.

Correct Answer : C

Answer Justification :

The office of Solicitor General has not been mentioned in the constitution

It is a statutory post created by an act of the Parliament

Hence the post may be abolished by an act of the Parliament.

Hence statement a is correct.

The Solicitor General is subordinate to the Attorney General of India ( a constitutional


office created under article 76)

instacourses.insightsonindia.com 12

https://upscmaterial.online/
Download From - https://upscmaterial.online/

.
Total Marks : 200.00
Test-24 (Subject)
( Insta Prelims Test Series 2023 )

He is the second highest law officer in the country

He works under the Attorney General of India

He also advises the government in legal matters.

Hence statement b is correct.

He is appointed by the President

The appointment is done on recommendation of the Appointments Committee of the


Cabinet chaired by the Prime minister

The appointment is done for a period of 3 years

Hence statement c is incorrect.

The Solicitor General represents the Government of India in the Supreme Court or High
Court in cases including suits, writ petitions, appeals and other proceedings in which the
government of India is a concerned party or is otherwise interested.

Hence statement d is correct.

12. Consider the following statements regarding Advocate General of the state
1. He enjoys all the privileges available to a member of the state legislature.
2. He is remuneration is determined by the governor of the state concerned.
3. He resigns when the council of minister resigns from office in a state.

Which of the statements given above is/are correct?


A. 2 and 3 only
B. 1 and 3 only
C. 1 only
D. 1, 2 and 3

Correct Answer : D

Answer Justification :

instacourses.insightsonindia.com 13

https://upscmaterial.online/
Download From - https://upscmaterial.online/

.
Total Marks : 200.00
Test-24 (Subject)
( Insta Prelims Test Series 2023 )

The constitution under article 165 provides for office of advocate general for States

He is the chief law officer of the government in the state

In performance of his official duties, he has the right to speak and take part in
proceedings of both houses of the state legislature or any committee of it of
which he may be named a member but without the right to vote

He enjoys all the privileges and immunities enjoyed by a member of the state
legislature

Hence statement 1 is correct.

The remuneration of the advocate general is not fixed by the constitution

He receives such remuneration as the governor may determine

Hence statement 2 is correct.

The term of office of advocate general is not fixed by the constitution

The constitution does not provide for procedure and grounds for his removal

He holds office during the pleasure of the governor and can be removed by him at any
time

He may also quit his office by submitting his resignation to the governor

Conventionally, he resigns when the government (Council of ministers) resigns or


is replaced as he is appointed on its advice.

Hence statement 3 is correct.

13. Consider the following statements


1. A presidential ordinance becomes inoperative if the Parliament, after it’s reassembly, does not
approve it within six weeks.

instacourses.insightsonindia.com 14

https://upscmaterial.online/
Download From - https://upscmaterial.online/

.
Total Marks : 200.00
Test-24 (Subject)
( Insta Prelims Test Series 2023 )

2. The governor can promulgate or withdraw an ordinance only on advise of state Council of
ministers.

Which of the statements given above is/are correct?


A. 1 only
B. 2 only
C. Both 1 and 2
D. Neither 1 nor 2

Correct Answer : C

Answer Justification :

Every ordinance issued by the president during the recess of the Parliament must be laid
before both the houses of the Parliament when it reassembles.

If the ordinance is approved by both the houses it becomes an act

But if Parliament takes no action at all, the ordinance ceases to operate on the expiry of 6
weeks from the reassembly of the Parliament.

Hence statement 1 is correct.

The ordinance making power of the governor is not a discretionary power.

He can promulgate or withdraw and ordinance only on advice of the Council of ministers
headed by the Chief minister.

Hence statement 2 is correct.

14. Consider the following statements regarding the Constitution of India


1. The constitution under article 27 provides that no fee can be levied by the State for control of
administration of a religious institution.
2. The origin of Directive Principles of State Policy can be traced to the 'Instrument of Instructions'
enumerated in the Government of India Act 1935.

Which of the statements given above is/are correct?


A. 1 only
B. 2 only
C. Both 1 and 2

instacourses.insightsonindia.com 15

https://upscmaterial.online/
Download From - https://upscmaterial.online/

.
Total Marks : 200.00
Test-24 (Subject)
( Insta Prelims Test Series 2023 )

D. Neither 1 nor 2

Correct Answer : D

Answer Justification :

Article 27 provides that the state cannot spend public money collected by way of tax for
maintenance or promotion of any particular religion

It prohibits only levy of a tax and not a fee

This is because the purpose of a fee is to control secular administration of religious


institutions and not promotion or maintenance of religion

Thus, a fee can be levied on pilgrims to provide them some special service or safety
measures.

Hence statement 1 is incorrect.

The Directive Principles of State Policy was borrowed by makers of Indian


constitution from Iris constitution of 1937 which in turn had copied the same
from the Spanish constitution.

The Directive Principles also resemble Instrument of Instructions enumerated in the


Government of India Act 1935.

These principles have been described by Dr BR Ambedkar as novel features of the


Indian Constitution.

Hence statement 2 is incorrect.

15. Consider the following statements


1. The Chief Commissioner’s Provinces under the British were constituted as union territories
under provisions of First Constitutional Amendment Act 1951.
2. The president has to revoke a proclamation of National emergency when a resolution passed by
a special majority of the Lok Sabha is presented to him for the same.

Which of the statements given above is/are correct?


A. 1 only

instacourses.insightsonindia.com 16

https://upscmaterial.online/
Download From - https://upscmaterial.online/

.
Total Marks : 200.00
Test-24 (Subject)
( Insta Prelims Test Series 2023 )

B. 2 only
C. Both 1 and 2
D. Neither 1 nor 2

Correct Answer : D

Answer Justification :

During the British rule certain areas were constituted as scheduled districts in 1874

Later they came to be known as chief commissioner's provinces

After independence, they were placed in the category of part C and part D States

th
In 1956, they were constituted as union territories by the 7 Constitutional Amendment
Act 1956 and the States Reorganisation Act 1956.

Hence statement 1 is incorrect.

A proclamation of National emergency maybe revoked by the president at any time by a


subsequent proclamation

Further, the president must revoke a proclamation of National emergency if a resolution is


passed for its disapproval by the Lok Sabha

The 44th Amendment Act of 1978 provided that when one-tenth of the total members of
the Lok Sabha give a written notice to the speaker, a special sitting of the house should
be held within 14 days for the purpose of considering a resolution disapproving
continuation of the proclamation

such a resolution can be adopted by a simple majority in the house.

Hence statement 2 is incorrect.

16. Consider the following statements


1. A Tribunal for the purpose of adjudication of disputes related to election to Parliament and state
legislature can be established both by Parliament and state legislatures.
2. A Bill passed by the state legislature endangering position of the state High Court should be

instacourses.insightsonindia.com 17

https://upscmaterial.online/
Download From - https://upscmaterial.online/

.
Total Marks : 200.00
Test-24 (Subject)
( Insta Prelims Test Series 2023 )

reserved for consideration of the president by the governor.

Which of the statements given above is/are correct?


A. 1 only
B. 2 only
C. Both 1 and 2
D. Neither 1 nor 2

Correct Answer : C

Answer Justification :

Article 323B authorizes the Parliament and state legislatures to established tribunals for
adjudication of disputes relating to the following matters :

Taxation

Foreign exchange

Import and export

Industry and labour

Land Reforms

Ceiling on urban property

Elections to the Parliament and state legislatures

Food stuff

Rent and tenancy right

Hence statement 1 is correct.

When a Bill passed by the state legislature is sent to the Governor for his consideration,
he is empowered to reserve the bill for consideration of the president

instacourses.insightsonindia.com 18

https://upscmaterial.online/
Download From - https://upscmaterial.online/

.
Total Marks : 200.00
Test-24 (Subject)
( Insta Prelims Test Series 2023 )

In one case such reservation is obligatory viz., when the bill passed by the state
legislature endangers position of the state High Court.

In addition, when a bill is against provisions of the Constitution, opposes Directive


principles, is against the larger interest of the country, deals with compulsory acquisition
of property under article 31A of the constitution is passed by the state legislature, the
governor can reserve the bill for Presidential assent.

Hence statement 2 is correct.

17. Consider the following statements regarding the Governor


1. Appointment of an individual as a governor of two States is a parliamentary convention and not
mentioned in the Constitution.
2. The Constitution provides that the chief minister of the state concerned should be consulted
while appointment of a governor is made by the president.
3. The allowances of the governor are determined by the parliament.

Which of the statements given above is/are correct?


A. 2 and 3 only
B. 1 only
C. 3 only
D. 1 and 2 only

Correct Answer : C

Answer Justification :

Articles 153 to 167 in part VI of the Constitution deal with the state executive consisting
of the governor, Chief minister, Council of ministers and advocate general of the state

The governor is appointed by the president

Usually there is a governor for each state but the 7th Constitutional Amendment
Act of 1956 facilitated appointment of the same person as a governor for two or
more States.

Hence statement 1 is incorrect.

The constitution provides only two qualifications for appointment of a person as a governor

instacourses.insightsonindia.com 19

https://upscmaterial.online/
Download From - https://upscmaterial.online/

.
Total Marks : 200.00
Test-24 (Subject)
( Insta Prelims Test Series 2023 )

He should be a citizen of India

He should have completed 35 years of age.

Two conventions have developed in this regard over years:

The governor should not belong to the state where he is appointed

While appointing the governor the president should consult Chief minister of the state
concerned.

However, both the conventions have been violated in some cases

Hence statement 2 is incorrect.

The constitution provides for certain conditions of the governor’s office the governor is
entitled without payment of rent to the use of his official residence

He is entitled to such emoluments, allowances and privileges as may be determined by


the parliament.

Those Emoluments and allowances cannot be diminished during his term of office.

Hence statement 3 is correct.

18. Consider the following statements


1. Certiorari can be rissued against a Tribunal for the purpose of transfer of a case pending with it
to a higher court.
2. Quo- Warranto can be issued only in case of a Constitutional authority.
3. Prohibition can be issued only against judicial and quasi-judicial authorities.

Which of the statements given above is/are correct?


A. 1 only
B. 2 and 3 only
C. 2 only
D. 1 and 3 only

Correct Answer : D

instacourses.insightsonindia.com 20

https://upscmaterial.online/
Download From - https://upscmaterial.online/

.
Total Marks : 200.00
Test-24 (Subject)
( Insta Prelims Test Series 2023 )

Answer Justification :

The writ of Certiorari means to be certified or to be informed.

It is issued by a higher Court to a Lower court or Tribunal either to transfer a case pending
with the later to itself or to squash the order of the later in a case.

It is basically issued in case of excess jurisdiction.

Hence statement 1 is correct.

Quo-Warranto means by what authority or warrant.

It can be issued by the court to enquire into legality of claim of a person to a public office.

It can be issued only in case of a substantive public office of a permanent


character created by a statute or by the constitution

it cannot be issued in cases of ministerial office or private office.

Hence statement 2 is incorrect.

Prohibition means to forbid

It is issued by a higher court to a Lower court or tribunal to prevent the later from
exceeding its jurisdiction or usurping a jurisdiction it does not have.

It can be issued only against judicial and quasi-judicial authorities

Administrative authorities, legislative bodies and private individuals are


outside its purview.

Hence statement 3 is correct.

19. Consider the following statements regarding the Delimitation Commission


1. It is appointed by the Election Commission of India for the purpose of redrawing boundaries of
Lok Sabha and state assembly seats.

instacourses.insightsonindia.com 21

https://upscmaterial.online/
Download From - https://upscmaterial.online/

.
Total Marks : 200.00
Test-24 (Subject)
( Insta Prelims Test Series 2023 )

2. The members are drawn from state election commissions and is chaired by the Chief Election
commissioner.
3. No order of the commission can be subjected to judicial review.

Which of the statements given above is/are correct?


A. 1 only
B. 3 only
C. 2 and 3 only
D. 1, 2 and 3

Correct Answer : B

Answer Justification :

Delimitation is the act of redrawing boundaries of Lok Sabha and state legislative
assembly seats to represent changes in population

The main objective of delimitation is to provide equal representation to equal segments of


a population.

Delimitation is carried out by an independent Delimitation Commission that is appointed


by the government of India under provisions of Delimitation Commission act

The commission is appointed by the President of India and works in collaboration with the
Election Commission of India.

Hence statement 1 is incorrect.

The commission is composed of

A retired Supreme Court judge

The Chief Election Commissioner of India and

Respective state election commissioners.

Hence statement 2 is incorrect.

The Delimitation commission is to work without any executive influence.

instacourses.insightsonindia.com 22

https://upscmaterial.online/
Download From - https://upscmaterial.online/

.
Total Marks : 200.00
Test-24 (Subject)
( Insta Prelims Test Series 2023 )

The Constitution mandates that the commission’s orders are final and cannot be
questioned before any Court as that would hold up an election indefinitely.

Hence statement 3 is correct.

20. Consider the following statements regarding the parliament


1. A Secret sitting of the Lok Sabha can be convened only on permission of the president.
2. The fundamental right to freedom of press regarding publication of true reports of
parliamentary procedents without permission of the Lok Sabha is not applicable for a secret
sitting.

Which of the statements given above is/ are correct?


A. 1 only
B. 2 only
C. Both 1 and 2
D. Neither 1 nor 2

Correct Answer : B

Answer Justification :

The speaker of the Lok Sabha is empowered to allow a secret sitting of the house

He can do so at the request of the Leader of the House.

When the house seats in secret, no stranger can be present in the chamber, lobby or
galleries except with permission of the speaker.

Hence statement 1 is incorrect.

Each house of the Parliament has the right to publish its reports, debates and proceedings
and also the right to prohibit others from publishing the same

The 44th Amendment Act of 1978 restored the freedom of press to publish true
reports of parliamentary proceedings without prior permission of the house

But this is not applicable in case of a secret sitting of the House

Hence statement 2 is correct.

instacourses.insightsonindia.com 23

https://upscmaterial.online/
Download From - https://upscmaterial.online/

.
Total Marks : 200.00
Test-24 (Subject)
( Insta Prelims Test Series 2023 )

21. Regarding Estimates Committee, which of the following statements is incorrect?

A. All members of the committee are from the Lok Sabha, whose duty is to examine
estimates included in the budget and suggest economies in public expenditure.
B. The members are elected in a way such that all parties get due representation in the
committee.
C. The Prime minister act as chairman of the committee.
D. The origin of the committee can be traced to the pre-independence period.

Correct Answer : C

Answer Justification :
The Estimates Committee consist of 30 members
All the members are selected from Lok Sabha only
The Rajya Sabha has no representation in it
The members are elected by the Lok Sabha every year from amongst its members.
The function of the committee is to examine estimates included in the budget and
suggest economies in public expenditure.

Hence statement a is correct.

The members of the committee are elected every year according to the principles of
proportional representation by means of single transferable vote.
In this way all parties get due representation in it.
The members hold office for a period of one year.

Hence statement b is correct.

No minister can be elected as a member of the committee


The chairman of the committee is appointed by the speaker from amongst its member
The chairman is selected from the ruling party.

Hence statement c is incorrect

The origin of the Estimates Committee can be traced to the standing financial committee
setup in 1921
The first Estimates Committee in the post-independence era was constituted in 1950 on
the recommendation of John Mathai, the then Finance minister.

Hence statement d is correct.

22. Regarding Comptroller and Auditor General of India, which of the following
statements is correct?

A. He holds office till pleasure of the President who also determines his service conditions.

instacourses.insightsonindia.com 24

https://upscmaterial.online/
Download From - https://upscmaterial.online/

.
Total Marks : 200.00
Test-24 (Subject)
( Insta Prelims Test Series 2023 )

B. He can be removed on the basis of a resolution passed by both houses of the Parliament
with special majority.
C. The administrative expenses of the office of CAG forms part of expenditure made from
the consolidated fund of India and is subjected to voting of the Parliament.
D. He is eligible for reappointment after retirement as an auditor in public corporations only.

Correct Answer : B

Answer Justification :
Article 148 of the constitution provides for an independent office of Comptroller and
Auditor General of India
He is appointed by the president
He holds office for a period of 6 years or up to 65 years of age whichever is
CAG does not hold office till the pleasure of the president though he is appointed by him
His salary, allowances and other service conditions are determined by the
Parliament
His salary is equal to that of a Supreme Court judge

Hence statement a is incorrect.

The CAG can be removed by the president on the same grounds and in same manner as a
judge of the supreme court
He can be removed by the president based on a resolution passed to that affect by both
houses of the Parliament with special majority
The removal can be made either on ground of proved misbehaviour or incapacity.

Hence statement b is correct.

The administrative expenses of the office of CAG including all salaries, allowances and
pensions of persons serving in that office are charged upon the consolidated fund of
India
Thus, they are not subject to vote of the Parliament.

Hence statement c is incorrect.

CAG is not eligible for further office either under the Government of India or any state
after he ceases to hold his office.

Hence statement d is incorrect.

23. Which of the following ensures independence of the office of the speaker of Lok
Sabha?
1. His rank in the Table of Precedence is same as that of the Chief Justice of India
2. He can decide the question of disqualification of a member of parliament
3. His salaries and allowances are charged on the Consolidated Fund of India

instacourses.insightsonindia.com 25

https://upscmaterial.online/
Download From - https://upscmaterial.online/

.
Total Marks : 200.00
Test-24 (Subject)
( Insta Prelims Test Series 2023 )

4. He is the final interpreter of the provisions of the Constitution of India within the Lok Sabha
5. He can exercise only a casting vote in case of a tie.
6. He can be removed by the Lok Sabha only by an absolute majority.

Select the correct answer using the codes given below


A. 1, 2, 4 and 5 only
B. 1, 3, 5 and 6 only
C. 2, 4, 5 and 6 only
D. All of the above

Correct Answer : B

Answer Justification :

The office of the speaker is vested with great prestige position and authority. The following
provisions ensure independent and impartiality of the office of the speaker:

He is provided with a security of tenure; he can be removed only by a resolution passed


by the Lok Sabha by an absolute majority. This motion for removal can be considered and
discussed only when it has the support of at least 50 members
His salaries and allowances are charged on the consolidated fund of India and not subject
to annual vote of the Parliament
His work and conduct cannot be discussed and criticized except on a substantive motion
His powers of regulating procedure or conducting business or maintaining order in the
house are not subject to jurisdiction of any Court
He cannot vote in the first instance; this makes his position impartial
His placed at the 7th rank along with the Chief Justice of India ; this means that he has a
higher rank than all the cabinet ministers except Prime minister or Deputy Prime minister.

Hence option B is correct.

24. Consider the following statements


1. A Remote Voting Machine is an electronic voting machine enabled to handle multiple
constituences from a single remote polling booth.
2. Electoral rolls for an Assembly constituency is prepared by the Chief Electoral Officer of a state.

Which of the statements given above is/are correct?


A. 1 only
B. 2 only
C. Both 1 and 2
D. Neither 1 nor 2

Correct Answer : A

instacourses.insightsonindia.com 26

https://upscmaterial.online/
Download From - https://upscmaterial.online/

.
Total Marks : 200.00
Test-24 (Subject)
( Insta Prelims Test Series 2023 )

Answer Justification :
Recently the Election Commission of India said that it is ready to use remote voting for
domestic migrants as a pilot project through newly devised Remote Voting Machines.
RVM prototype will enable a voter who is listed in constituencies, to exercise their voting
rights from a single machine
The multi-constituency remote EVM can handle up to 72 constituencies from a single
remote polling booth.

Hence statement 1 is correct.

The Electoral Registration Officer is responsible for the preparation of electoral


rolls for a parliamentary/ Assembly constituency.
The Election Commission of India appoints an officer of the government or local authority
as Electoral Registration Officer in consultation with the state/ UT government.

Hence statement 2 is incorrect.

25. Consider the following statements


1. Article 394A provided for publication of an authoritative text of the Constitution of India in Hindi
language in the Gazzette of India.
2. The 58th Constitutional Amendment Act of 1987 provided that it is not mandatory to provide a
Hindi translation of every amendment of the Constitution.

Which of the statements given above is/are correct?


A. 1 only
B. 2 only
C. Both 1 and 2
D. Neither 1 nor 2

Correct Answer : A

Answer Justification :
Originally the constitution did not provide for an authoritative text of the constitution in
Hindi language
The 58th Constitutional Amendment Act of 1987 inserted article 394A in part XXII
of the constitution
It provided that the president shall, under his authority, provide for translation of the
Constitution in the Hindi language
In pursuance of this article, the president published in the Gazette of India, an
authoritative text of the Constitution in Hindi language.

Hence statement 1 is correct.

The 58th Constitutional Amendment Act of 1987 provided that the president
shall also cause to be published under his authority the translation in Hindi of

instacourses.insightsonindia.com 27

https://upscmaterial.online/
Download From - https://upscmaterial.online/

.
Total Marks : 200.00
Test-24 (Subject)
( Insta Prelims Test Series 2023 )

every amendment of the Constitution made in English

Hence statement 2 is incorrect

26. Consider the following statements regarding Public Accounts Committee (PAC)
1. It was initially established as a standing financial committee in post independence period on
recommendation of Krishna Menon committee.
2. All reports audited by the Comptroller and Auditor General of India related to appropriation
accounts are scrutinised by the committee.
3. Although the Comptroller and Auditor General is not a member of the committee, he is to assist
the committee in exercise of examining of reports submitted by him.

Which of the statements given above is/are correct?


A. 1 and 3 only
B. 1 and 2 only
C. 2 only
D. 2 and 3 only

Correct Answer : D

Answer Justification :
PAC was set up 1921 under provisions of Government of India Act 1919 and has since
been in existence.
The committee on public undertakings was created in 1964 on recommendation of the
Krishna Menon committee.

Hence statement 1 is incorrect.

The function of the committee is to examine the annual audit reports of CAG which are
laid before the Parliament by the president.
The committee examines public expenditure not from legal and formal point of view to
discover technical irregularities but also from point of view of economy, wisdom and
propriety to bring out the cases of waste, loss, corruption inefficiency and so on.

Hence statement 2 is correct.

The main functions of the Committee is to examine the annual audit reports of CAG.
It also examines accounts of state corporations, trading concerns and the audit reports of
CAG on them, accounts of autonomous and semi-autonomous bodies, money spent on
any service during a financial year in excess of amount granted by the Lok Sabha for that
purpose.
In fulfilment of the mentioned specified functions, the committee is assisted by the CAG.

Hence statement 3 is correct.

instacourses.insightsonindia.com 28

https://upscmaterial.online/
Download From - https://upscmaterial.online/

.
Total Marks : 200.00
Test-24 (Subject)
( Insta Prelims Test Series 2023 )

27. Consider the following statements regarding Municipal Corporation


1. The Municipal Council consists of both directly elected and nominated members.
2. Decisions related to public works and their implementation are carried on by standing
committees of the corporation.
3. The chief executive authority of the corporation is vested in the Mayor who is elected for a
period of one year.

Which of the statements given above is/are correct?


A. 1 and 2 only
B. 1 and 3 only
C. 2 only
D. 2 and 3 only

Correct Answer : A

Answer Justification :
A Municipal Corporation has three authorities viz., the council, standing committee and
the commissioner
The council is the deliberative and legislative wing of the corporation
It consists of the councillors directly elected by the people as well as few
nominated persons having knowledge or experience of municipal
administration

Hence statement 1 is correct.

The standing committees of the corporation are created to facilitate working of the
Council which is too large in size
They deal with public works, education, health, taxation, finance and so on
They take decisions in their fields.

Hence statement 2 is correct.

Municipal Council is headed by a Mayor who is elected in a majority of the states for a 1
year term
He is an ornamental figure and formal head of the corporation whose main function is to
preside over meetings of the Council
The Municipal commissioner is responsible for implementation of the decisions
taken by the Council and its standing committees
He is the chief executive authority of the corporation
He is appointed by the state government and is generally a member of the IAS.

Hence statement 3 is incorrect.

28. Consider the following statements regarding Panel of chairpersons of Lok Sabha

instacourses.insightsonindia.com 29

https://upscmaterial.online/
Download From - https://upscmaterial.online/

.
Total Marks : 200.00
Test-24 (Subject)
( Insta Prelims Test Series 2023 )

1. They are nominated by the speaker from amongst the members of the House.
2. Anyone of them is to preside over the House when the office of Speaker or Deputy Speaker falls
vacant.
3. A member of this panel is selected, and appointed as the Speaker Pro Tem by the president.

Which of the statements given above is/are correct?


A. 2 only
B. 1 only
C. 2 and 3 only
D. 1 and 2 only

Correct Answer : B

Answer Justification :
Under the rules of Lok Sabha, the speaker nominates, from amongst the members, a
panel of not more than 10 chairpersons
Any of them can preside over the house in absence of speaker or deputy
speaker
He has the same power as the speaker when so presiding
He holds office until a new panel of chairpersons is nominated.

Hence statement 1 is correct.

It must be emphasized that a member of the panel of chairpersons cannot preside


over the Lok Sabha when the office of speaker or deputy speaker is vacant
During such time, the duties of the speaker are to be performed by such member of the
house as the President may appoint for the purpose.

Hence statement 2 is incorrect.

Under the constitution, the speaker of the last Lok Sabha vacates his office immediately
before first meeting of the newly elected Lok Sabha
Hence the president appoints a member of the Lok Sabha as the speaker Pro Tem.
Usually, the seniormost member of the house is selected for this.

Hence statement 3 is incorrect.

29. Consider the following statements


1. A Provision related to elections to the state legislature can be made by the Parliament only.
2. An election petition challenging an election to the state legislature can be tried by the supreme
court only.

Which of the statements given above is/are correct?


A. 1 only
B. 2 only

instacourses.insightsonindia.com 30

https://upscmaterial.online/
Download From - https://upscmaterial.online/

.
Total Marks : 200.00
Test-24 (Subject)
( Insta Prelims Test Series 2023 )

C. Both 1 and 2
D. Neither 1 nor 2

Correct Answer : D

Answer Justification :
Article 324 to 329 in part XV of the Constitution makes certain provisions with respect to
electoral system in the country.
The state legislatures can make provisions with respect to all matters relating to elections
to the state legislatures including preparation of electoral role and all other matter
necessary for securing their due constitution
But they can make provision only for those matters which are not covered by
the parliament
In other words, state legislatures can only supplement the parliamentary law
and cannot override it.

Hence statement 1 is incorrect.

The constitution provides that no election to the Parliament or state legislature is to be


questioned except by an election petition presented to such authority and in such manner
as provided by the appropriate legislature
Since 1966 election petitions are triable by high courts alone
However, appellate jurisdiction lies with the supreme court alone.

Hence statement 2 is incorrect.

30. Consider the following statements


1. By ‘Adjournment’, the presiding officer of a house suspends work in a sitting of the House for a
specified time.
2. The power of ' Adjournment sine die' and ‘Prorogation’ lies with the president of India.
3. Both ‘Adjournment’ and 'Prorogation' has no effect on bills pending before a House of the
Parliament.

Which of the statements given above his correct?


A. 1 and 3 only
B. 2 and 3 only
C. 1 and 2 only
D. 1 only

Correct Answer : A

Answer Justification :
Adjournment terminates a sitting of a house
It suspends work in a sitting for a specified time which may be hours, days or weeks

instacourses.insightsonindia.com 31

https://upscmaterial.online/
Download From - https://upscmaterial.online/

.
Total Marks : 200.00
Test-24 (Subject)
( Insta Prelims Test Series 2023 )

The power of adjournment lies with presiding officer of a house of the


Parliament.

Hence statement 1 is correct.

Adjournment sine die means terminating a sitting of the Parliament for an indefinite
period
The power of Adjournment sine die lies with presiding officer of the house.
On the other hand, the presiding officer declares the house Adjournment sine die when
business of a session is completed
Within the next few days, the president issues a notification for prorogation of the house.

Hence statement 2 is incorrect.

Both Adjournment and Prorogation does not affect bills or any other business
pending before the house.
However, in case of prorogation, all pending notices lapse and fresh notices have to be
given for the next session.

Hence statement 3 is correct.

31. Regarding Cabinet Committees, which of the following statements is incorrect?

A. They are extra-Constitutional bodies set up under the Rules of Business


B. Both cabinet and non cabinet ministers can be members of these committees.
C. They can either be permanent or temporary in nature.
D. All cabinet committees are set up and headed by the Speaker of the Lok Sabha.

Correct Answer : D

Answer Justification :
Cabinet committees are extra-constitutional in nature.
They are established under Rules of Business.
These are organisational device to reduce enormous workload of the cabinet
They facilitate in-depth examination of policy issues and effective coordination.

Hence statement a is correct.

Usually, a cabinet committee consists of three to eight members; they usually include
only cabinet ministers.
But non cabinet ministers can also become member of any type of cabinet committee.

Hence statement b is correct.

Cabinet committees are of two types- Standing and Ad hoc.


Standing committees are permanent in nature while Ad hoc ones are temporary in nature.

instacourses.insightsonindia.com 32

https://upscmaterial.online/
Download From - https://upscmaterial.online/

.
Total Marks : 200.00
Test-24 (Subject)
( Insta Prelims Test Series 2023 )

The ad hoc committees are constituted from time to time to deal with special issues and
are disbanded after the task is completed.
Cabinet committees help in sorting out of issues and formulating proposals for
consideration of the cabinet also take certain decisions the cabinet can review their
decisions.

Hence statement c is correct.

Cabinet committees are set up by the Prime Minister according to exigencies of time and
requirements of the situation
Their number, composition and nomenclature vary from time to time.
These committees are mostly headed by the Prime Minister.
But sometimes, other cabinet minister, particularly Home Minister or the
Finance Minister can also act as their chairman.
But in case the Prime Minister is a member of a committee, he presides over it.

Hence statement d is incorrect

32. Consider the following statements regarding Autonomous Districts


1. The constitution empowers the governor to organised and define boundaries of such districts.
2. An autonomous district is created for a particular tribal group.
3. The constitution empowers the President to constitute a Commission to examine administrative
system in autonomous districts.

Which of the statements given above is/are correct?


A. 1 and 3 only
B. 2 only
C. 1 only
D. 1 and 2 only

Correct Answer : C

Answer Justification :
Under the Sixth schedule of the constitution, the tribal areas in the four states of Assam,
Meghalaya, Tripura and Mizoram have been constituted as autonomous districts
The Governor is empowered to organise and reorganise the autonomous districts
Thus, he can increase or decrease their areas or change their names or define
their boundaries.

Hence statement 1 is correct.

There may be different tribes in an autonomous district


In such case, the governor can divide the district into several autonomous regions.

Hence statement 2 is incorrect.

instacourses.insightsonindia.com 33

https://upscmaterial.online/
Download From - https://upscmaterial.online/

.
Total Marks : 200.00
Test-24 (Subject)
( Insta Prelims Test Series 2023 )

The constitution provides that the governor can appoint a Commission to examine
and report on any matter relating to the administration of the autonomous
districts or regions.
He may dissolve a district or regional council on recommendation of the commission.

Hence statement 3 is incorrect.

33. Consider the following statements


1. No authority can question a decision of the president regarding age of a High Court judge.
2. An individual who has served a judicial office or has been an advocate of a High Court, both for
more than five years, is eligible to be appointed as a high court judge.

Which of the statements given above is/are correct?


A. 1 only
B. 2 only
C. Both 1 and 2
D. Neither 1 nor 2

Correct Answer : A

Answer Justification :
The constitution has not fixed the tenure of a High Court judge
He holds office until he attains the age of 62 years
Any question regarding his age is to be decided by the president after
consulting the Chief Justice of India and the decision of the president is final.

Hence statement 1 is correct.

A person to be appointed as a judge of the high court should possess the following qualifications
:

He should be a citizen of India


He should have held a judicial office in the territory of India for 10 years
He should have been an advocate of a High Court or High Courts in succession for 10
years

Hence statement 2 is incorrect.

34. Which of the following has replaced the British Privy Council in the post-
independence period?

A. Reserve Bank of India


B. Supreme Court of India

instacourses.insightsonindia.com 34

https://upscmaterial.online/
Download From - https://upscmaterial.online/

.
Total Marks : 200.00
Test-24 (Subject)
( Insta Prelims Test Series 2023 )

C. Office of President
D. Council of ministers

Correct Answer : B

Answer Justification :
The Supreme Court has replaced the British Privy Council as the highest court
of appeal.
Before 1950 the British Privy Council had the jurisdiction to hear appeals from India
The Supreme Court of India was inaugurated in January 28, 1950
It succeeded the Federal Court of India which was established under the Government of
India Act 1935
Articles 124 to 147 in part V of the Constitution deals with organisation, independence,
jurisdiction, powers and procedure of the supreme court; the parliament is also authorised
to regulate them.

Hence option B is correct

35. Consider the following statements regarding Municipalities


1. The state legislature is authorised to provide for representation of members of Lok Sabha
representing constituencies that comprise wholly or partly the municipal area
2. The territorial area as well as composition of a ward committee are decided by the Governor.
3. The constitution provides that all persons qualified to be chosen to the state legislative Council
are qualified for being a member of municipality.

Which of the statements given above is/are correct?


A. 2 only
B. 1 and 3 only
C. 1 only
D. 1, 2 and 3

Correct Answer : A

Answer Justification :
The 74th Amendment Act of 1992 provides for composition of a municipality. All members
of municipality are elected directly by the people of the Municipal area.
Additionally, the state legislature may provide for the manner of election of the
chairperson of the municipality.
It may also provide for representation of the members of Lok Sabha and state legislative
assembly representing constituencies that comprise wholly or partly the municipal area.

Hence statement 1 is correct.

Ward committees are constituted consisting of one or more wards within the territorial

instacourses.insightsonindia.com 35

https://upscmaterial.online/
Download From - https://upscmaterial.online/

.
Total Marks : 200.00
Test-24 (Subject)
( Insta Prelims Test Series 2023 )

area of a municipality having population of 3 lakh or more.


The state legislature can make provision regarding composition and territorial
area of ward committees and the manner in which such a committee shall be
filled.

Hence statement 2 is incorrect.

article 243V lays down that all persons who are qualified to be chosen to the state
legislature shall be qualified for being a member of the municipality.
But there is an important difference; persons who have attained the age of 21
years will be eligible to be a member of municipality but for being elected to
the state Legislative Assembly, a person must have attained the age of 25
years and 30 years for Legislative Council.

Hence statement 3 is incorrect.

36. Consider the following statements regarding the Basic Exchange and Cooperation
Agreement (BECA):
1. It allows India to get real-time access to American geospatial intelligence enhancing the
accuracy of armed drones.
2. It allows the militaries of the United States and India to replenish from each other’s bases.
3. It allows the United States to provide India with encrypted communications systems during
times of both peace and war.

Which of the statements given above is/are correct?


A. 1 only
B. 1 and 2 only
C. 2 and 3 only
D. 1, 2 and 3

Correct Answer : A

Answer Justification :

India and the United States signed the Basic Exchange and Cooperation Agreement (BECA),
which, along with the two agreements signed earlier:

The Logistics Exchange Memorandum of Agreement (LEMOA) and


The Communications Compatibility and Security Agreement (COMCASA).

These complete a troika of “foundational pacts” for deep military cooperation between the two
countries.

Statement 1 is correct: BECA will help India get real-time access to American geospatial
intelligence that will enhance the accuracy of automated systems and weapons like missiles

instacourses.insightsonindia.com 36

https://upscmaterial.online/
Download From - https://upscmaterial.online/

.
Total Marks : 200.00
Test-24 (Subject)
( Insta Prelims Test Series 2023 )

and armed drones.

Through the sharing of information on maps and satellite images, it will help India access
topographical and aeronautical data, and advanced products that will aid in navigation and
targeting.

BECA will provide Indian military systems with a high-quality GPS to navigate missiles with real-
time intelligence to precisely target the adversary.

Statement 2 is not correct: LEMOA was the first of the three pacts to be signed in August
2016. LEMOA allows the militaries of the US and India to replenish from each other’s bases, and
access supplies, spare parts and services from each other’s land facilities, air bases, and ports.

Statement 3 is not correct: The COMCASA pact allows the US to provide India with its
encrypted communications equipment and systems so that Indian and US military commanders,
and the aircraft and ships of the two countries, can communicate through secure networks
during times of both peace and war.

Source:
https://indianexpress.com/article/explained/beca-india-us-trade-agreements-rajnath-
singh-mike-pompeo-6906637/

37. The Oslo Peace Accord sometimes mentioned in news was originally signed between-

A. Greece and Turkey


B. Ukraine and Russia
C. Armenia and Azerbaijan
D. Palestine and Israel

Correct Answer : D

Answer Justification :

Option (d) is correct: The “historic” Oslo Peace accord was signed between Israel and
Palestine.

A supplementary accord, Oslo II, was signed in 1995 dividing the West Bank into three
contiguous regions.
The signing ceremony of the Oslo accord in 1993 took place in the White House in
Washington.
For the first time, the Israeli Prime Minister, Yitzhak Rabin, and the Palestinian leader,
Yasser Arafat, shook hands under the benign gaze of United States President Bill Clinton.
It was the first public handshake by an Israeli and a Palestinian leader.
The accords were named after the Norwegian capital Oslo, where most of the secret
negotiations between Israel and the Palestine Liberation Organisation (PLO) took place

instacourses.insightsonindia.com 37

https://upscmaterial.online/
Download From - https://upscmaterial.online/

.
Total Marks : 200.00
Test-24 (Subject)
( Insta Prelims Test Series 2023 )

under U.S. supervision.

Source: https://frontline.thehindu.com/world-affairs/article25037186.ece

38. The key targets to be achieved under Immunization Agenda 2030 are-
1. Halving the number of children completely missing out on vaccines.
2. Achieving 90% coverage for essential vaccines given in adolescence.
3. Achieving full immunization coverage for all pregnant women.

Which of the statements given above is/are correct?


A. 1 only
B. 1 and 2 only
C. 2 and 3 only
D. 1, 2 and 3

Correct Answer : B

Answer Justification :

Immunisation Agenda 2030/IA2030 is an ambitious global strategy to maximize the lifesaving


impact of vaccines that, if fully implemented, will save 50 million lives over the next decade.

The Targets

Targets to be achieved by 2030 include:

Statement 1 is correct: Halving the number of children completely missing out on


vaccines
Statement 2 is correct: Achieving 90% coverage for essential vaccines given in
childhood and adolescence
Completing 500 national or subnational introductions of new or under-utilized vaccines-
such as those for COVID-19, rotavirus, or human papillomavirus (HPV).

Statement 3 is not correct: The ultimate goal of Mission Indradhanush is to ensure full
immunization with all available vaccines for children up to two years of age and pregnant
women.

Source:
https://www.who.int/teams/immunization-vaccines-and-biologicals/strategies/ia2030/
explaining-the-immunization-agenda-2030

https://www.nhp.gov.in/mission-indradhanush1_pg

instacourses.insightsonindia.com 38

https://upscmaterial.online/
Download From - https://upscmaterial.online/

.
Total Marks : 200.00
Test-24 (Subject)
( Insta Prelims Test Series 2023 )

39. Consider the following statements:


The Kalapani area is the largest territorial dispute between Nepal and India.
1. It is located in the easternmost corner of Uttarakhand’s Pithoragarh district.
2. It shares a border on the north with the Tibet Autonomous Region of China and Nepal in the
east.

Which of the statements given above is/are correct?


A. 1 only
B. 2 only
C. Both 1 and 2
D. Neither 1 nor 2

Correct Answer : C

Answer Justification :

Statement 1 is correct: Kalapani is a region located in the easternmost corner of


Uttarakhand’s Pithoragarh district.

The region resembles a slice of cake wedged in between Limpiyadhura, Lipulekh and Kalapani.

The area is in India’s control but Nepal claims the region because of historical and cartographic
reasons.

Statement 2 is correct: It shares a border on the north with the Tibet Autonomous Region of
China and Nepal in the east and south.

The area is the largest territorial dispute between Nepal and India consisting of at least 37,000
hectares of land in the High Himalayas.

The Kalapani region derives its name from the river Kali.

Nepal’s claims to the region is based on this river as it became the marker of the boundary of
the kingdom of Nepal following the Treaty of Sugauli signed between the Gurkha rulers of
Kathmandu and the East India Company after the Gurkha War/Anglo-Nepal War (1814-16).

According to the treaty, Nepal lost the regions of Kumaon-Garhwal in the west and Sikkim in the
east.

Source:
https://www.thehindu.com/news/national/why-are-india-and-nepal-fighting-over-kala
pani/article31660401.ece

40. The Minsk Group was created by the Organisation for Security and Cooperation in
Europe in the context of-

instacourses.insightsonindia.com 39

https://upscmaterial.online/
Download From - https://upscmaterial.online/

.
Total Marks : 200.00
Test-24 (Subject)
( Insta Prelims Test Series 2023 )

A. Daraa Insurgency
B. Nagorno-Karabakh Conflict
C. Nigerian bandit Conflict
D. Conflict of Rakhine State

Correct Answer : B

Answer Justification :

Option (b) is correct: The Minsk Group was created by the Organisation for Security and
Cooperation in Europe (OSCE) to facilitate talks between Armenia and Azerbaijan to find a
peaceful solution to the Nagorno-Karabakh conflict.

The Minsk Group was co-chaired by Russia, the United States, and France.

The “package deal” proposal of 1997 envisaged simultaneous removal of Armenian forces from
occupied areas and the determination of Nagorno-Karabakh’s status.

The role of the Minsk Group declined during the 2020 war between the two countries, as other
negotiating groups entered the scene.

Source:
https://www.thehindu.com/news/international/what-is-the-nagorno-karabakh-conflict
-explained/article65459587.ece

41. With reference to the Blue Dot Network, consider the following statements:
1. It is a part of the Act East Policy of India.
2. It aims to counter the Chinese Belt and Road Initiative.
3. It was jointly launched by Japan, India and Australia.

Which of the statements given above is/are not correct?


A. 1 only
B. 2 and 3 only
C. 1 and 3 only
D. 1, 2 and 3

Correct Answer : C

Answer Justification :

Statement 1 is not correct: The proposal for the Blue Dot network is part of the US’s Indo-
Pacific strategy,

Statement 2 is correct: It is aimed at countering Chinese President’s ambitious BRI.

instacourses.insightsonindia.com 40

https://upscmaterial.online/
Download From - https://upscmaterial.online/

.
Total Marks : 200.00
Test-24 (Subject)
( Insta Prelims Test Series 2023 )

Statement 3 is not correct: Led by the US’s International Development Finance Corporation
(DFC), the Blue Dot network was jointly launched by the US, Japan (Japanese Bank for
International Cooperation) and Australia (Department of Foreign Affairs and Trade) in November
2019 on the side lines of the 35th ASEAN Summit in Thailand.

It is meant to be a multi-stakeholder initiative that aims to bring governments, the private


sector and civil society together to promote “high quality, trusted standards for global
infrastructure development”.

Source:
https://indianexpress.com/article/explained/explained-what-is-the-blue-dot-network-
on-the-table-during-trump-visit-to-india-6286524/

42. The Copernicus Programme sometimes talked about in news is associated with-

A. Human exploration of the Moon


B. Earth observation programme
C. Exploration of Mars’ atmosphere
D. Exploration of Jupiter and its composition

Correct Answer : B

Answer Justification :

Option (b) is correct: Copernicus is the most ambitious Earth observation programme to
date.

It provides accurate, timely and easily accessible information to improve the


management of the environment, understand and mitigate the effects of climate change
and ensure civil security.
It builds and manages a fleet of Earth-observing satellites called Sentinel. Eight missions
have been launched to date, providing data about changes occurring to the oceans, land
and atmosphere of our planet.
This initiative is headed by the European Commission (EC) in partnership with the
European Space Agency (ESA).
ESA coordinates the delivery of data from upwards of 30 satellites.
The EC, acting on behalf of the European Union, is responsible for the overall initiative,
setting requirements and managing the services.

Source: https://www.space.com/copernicus-program

https://www.esa.int/Applications/Observing_the_Earth/Copernicus/Europe_s_Copernic
us_programme

instacourses.insightsonindia.com 41

https://upscmaterial.online/
Download From - https://upscmaterial.online/

.
Total Marks : 200.00
Test-24 (Subject)
( Insta Prelims Test Series 2023 )

43. Consider the following statements regarding the Wassenaar Arrangement (WA):
1. The countries in this arrangement subscribe to arms export controls similar to the Nuclear
Suppliers Group.
2. All the permanent members of the United Nations Security Council are signatories of the WA.

Which of the statements given above is/are correct?


A. 1 only
B. 2 only
C. Both 1 and 2
D. Neither 1 nor 2

Correct Answer : A

Answer Justification :

Statement 1 is correct: The Wassenaar Arrangement is an elite club of countries which


subscribe to arms export controls, similar to the Nuclear Suppliers Group and the Missile
Technology Control Regime.

The body came into being in 1996 to succeed the Cold War-era Coordinating Committee for
Multilateral Export Controls.

The name comes from Wassenaar, a suburb of The Hague, where the agreement to start such a
multi-lateral cooperation was reached in 1995.

Statement 2 is not correct: The WA has 42 members, the latest entrant being India. With the
exception of China, all the other permanent members of the U.N. Security Council are
signatories of the WA, which is headquartered in Vienna.

According to the WA website, the goal of the Arrangement is to "promote transparency and
greater responsibility in transfers of conventional arms and dual-use goods and technologies".

Participants are required to "ensure that transfers of these items do not contribute to the
development or enhancement of military capabilities which undermine the goal".

Source:
https://www.thehindu.com/news/international/what-is-wassenaar-arrangement/articl
e61847918.ece

44. The Ashgabat Agreement sometimes mentioned in news is a/an-

A. Agreement primarily focussing on Yemen’s humanitarian conditions.


B. Agreement on Economic Cooperation between India and the member states of the
Cooperation Council for the Arab States.
C. Agreement creating integrated multilateral trading system encompassing the General

instacourses.insightsonindia.com 42

https://upscmaterial.online/
Download From - https://upscmaterial.online/

.
Total Marks : 200.00
Test-24 (Subject)
( Insta Prelims Test Series 2023 )

Agreement on Tariffs and Trade.


D. Agreement that envisages the facilitation of transit and transportation of goods between
Central Asia and the Persian Gulf.

Correct Answer : D

Answer Justification :

Option (d) is correct: The Ashgabat Agreement is a move that strengthens trade ties between
India and the Eurasian region.

The decision paves the way for an international transport and transit corridor facilitating
transportation of goods between Central Asia and the Persian Gulf.
The Indian Cabinet has acceded to the Ashgabat Agreement. Oman, Iran, Turkmenistan
and Uzbekistan are the founding members of the Ashgabat Agreement, and Kazakhstan
joined it subsequently.
The decision is a big step towards enabling India to utilise the existing transport and
transit corridor to facilitate trade and commercial interaction with the Eurasian region.
It would synchronise with India’s efforts to implement the International North South
Transport Corridor (INSTC) for enhanced connectivity, he added.
India’s intention to accede to the Ashgabat Agreement would now be conveyed to
Turkmenistan, which is the Depository State.
India has become party to the Agreement after the consent of the founding members.

Source:
https://www.thehindu.com/opinion/op-ed/indias-central-asian-outreach/article371546
09.ece

Ashgabat Agreement gets Cabinet nod - The Hindu BusinessLine

45. The Rwandan Genocide recently talked about was a consequence of-

A. The troubled relationship between majority Hutus and minority Tutsis.


B. The conflict in Darfur that was a part of an ethnic cleansing campaign.
C. The migration of Nigerian Muslims from northern Nigeria to the Niger Delta.
D. The civil conflict involving the ethno-regional militia of Ethiopia’s Tigray.

Correct Answer : A

Answer Justification :

Option (a) is correct: The 1994 Rwandan massacre in which about 800,000 people, mostly
ethnic Tutsis, were killed.

instacourses.insightsonindia.com 43

https://upscmaterial.online/
Download From - https://upscmaterial.online/

.
Total Marks : 200.00
Test-24 (Subject)
( Insta Prelims Test Series 2023 )

The majority Hutus and minority Tutsis have had a troubled relationship in Rwanda that
goes back to the German and Belgian colonial period.
Colonialists ruled Rwanda through the Tutsi monarchy.
Tutsis were appointed as local administrative chiefs and the ethnic minority enjoyed
relatively better educational and employment opportunities, which led to widespread
resentment among the majority Hutus.
In 1959, Rwanda saw violent riots led by Hutus in which some 20,000 Tutsis were killed
and many more were displaced.
Since then, Rwanda had been controlled by Hutus, until their genocidal regime was
toppled by the Rwandan Patriotic Front (RPF) in 1994.
The killings were a pre-planned extermination campaign. The militias, with support from
the government, launched a premeditated violent campaign on April 7, aimed at
eliminating the entire Tutsi communities.

Source:
https://www.thehindu.com/news/international/the-hindu-explains-frances-role-in-the-
rwandan-genocide/article34666553.ece

46. Consider the following:


1. Exercise Garuda-VII
2. Exercise Cyclone-I
3. Exercise Mitrashakti
4. Exercise Maitree

Which of the above mentioned defence exercises was/were conducted between India and its
immediate neighbours?
A. 1 only
B. 3 only
C. 3 and 4 only
D. 1, 2 and 4 only

Correct Answer : B

Answer Justification :

Statement 1 is not correct: Garuda VII is a joint air force exercise between India and France.

The two countries began the Garuda series in India in 2003; Garuda VII was the seventh
iteration.

Statement 2 is not correct: The Exercise Cyclone-I is the first-ever joint exercise between the
special forces of the Indian Army and the Egyptian Army. The joint exercise aims to bolster
defence co-operation between the two nations.

The focus is also on sharing professional skills and interoperability of Special Forces in desert

instacourses.insightsonindia.com 44

https://upscmaterial.online/
Download From - https://upscmaterial.online/

.
Total Marks : 200.00
Test-24 (Subject)
( Insta Prelims Test Series 2023 )

terrain while undertaking reconnaissance, counter-terrorism, raids, and other special


operations.

Statement 3 is correct: Exercise Mitra Shakti, based on counter insurgency and counter
terrorism operations in semi urban terrain is the largest bilateral exercise being undertaken by
the Sri Lankan Army and it forms a major part of India and Sri Lanka’s growing defence
partnership.

Statement 4 is not correct: Exercise MAITREE is an annual training event which is being
conducted alternatively in Thailand and India since 2006.

Source:
https://www.financialexpress.com/photos/defence-gallery/2955176/cyclone-i-india-eg
ypt-special-forces-hold-joint-exercise-in-jaisalmer-photos/2/

https://pib.gov.in/PressReleaseIframePage.aspx?PRID=1878228

Press Information Bureau (pib.gov.in)

47. Operation Ganga Initiative was launched in context of-

A. Large-scale evacuations of Indian citizens from Afghanistan.


B. Sending humanitarian relief to Donetsk region of Ukraine.
C. Bringing back the Indian citizens stranded in Ukraine.
D. Relief operation in Earthquake hit Nepal by India.

Correct Answer : C

Answer Justification :

Option (a) is not correct: India's evacuation mission from Afghanistan was named as
'Operation Devi Shakti'.

Option (b) is not correct: Under 'Operation Ganga', India has successfully brought back more
than a 1,000 of its nationals from Ukraine.

It has set up 24×7 control centres to assist in the evacuation of Indians through the border
crossing points with Hungary, Poland, Romania and Slovak Republic.

However, humanitarian relief was not included under the operation.

Option (c) is correct: ‘Operation Ganga’ is the initiative launched by the government of India
to bring back Indians stranded in Ukraine.

Option (d) is not correct: The Indian Army through the Operation Maitri provided aid to
affected population and areas in close coordination and cooperation with the Nepal Army.

instacourses.insightsonindia.com 45

https://upscmaterial.online/
Download From - https://upscmaterial.online/

.
Total Marks : 200.00
Test-24 (Subject)
( Insta Prelims Test Series 2023 )

Source:
https://indianexpress.com/article/india/russia-ukraine-crisis-india-evacuation-operati
on-ganga-7794534/

https://www.newindianexpress.com/nation/2021/aug/24/indias-evacuation-mission-fr
om-afghanistan-named-operation-devi-shakti-2349109.html

48. With reference to the Indo-Pacific Economic Framework, consider the following
statements:
1. It broadly rests on supply chain resilience and decarbonisation as one of its pillars but does not
cover the anti-corruption measures.
2. Countries are free to join initiatives under any of the stipulated pillars but are expected to
adhere to all commitments once they enrol.

Which of the statements given above is/are correct?


A. 1 only
B. 2 only
C. Both 1 and 2
D. Neither 1 nor 2

Correct Answer : B

Answer Justification :

Indo-Pacific Economic Framework for Prosperity (IPEF) is a U.S.-led economic grouping


comprising 12 countries. These include Australia, Brunei, India, Indonesia, Japan, South Korea,
Malaysia, New Zealand, the Philippines, Singapore, Thailand and Vietnam.

Together, these countries account for 40 per cent of the global GDP.

Statement 1 is not correct: The economic framework broadly rests on four pillars: trade,
supply chain resilience, clean energy and decarbonisation, and taxes and anti-corruption
measures.

Statement 2 is correct: Countries are free to join (or not join) initiatives under any of the
stipulated pillars but are expected to adhere to all commitments once they enrol.

Negotiations slated to begin after the launch are meant to determine and list the provisions
under each pillar and open the floor for countries to choose their ‘commitments’.

Source: Explained | What is the Indo-Pacific Economic Framework for Prosperity? -


The Hindu

instacourses.insightsonindia.com 46

https://upscmaterial.online/
Download From - https://upscmaterial.online/

.
Total Marks : 200.00
Test-24 (Subject)
( Insta Prelims Test Series 2023 )

49. Which one of the following Organisations releases the Global Economic Prospects
Report?

A. The World Economic Forum


B. The International Monetary Fund
C. The World Bank Group
D. The Organisation for Economic Co-operation and Development

Correct Answer : C

Answer Justification :

Option (c) is correct: Global growth is slowing sharply in the face of elevated inflation, higher
interest rates, reduced investment, and disruptions caused by Russia’s invasion of Ukraine,
according to the World Bank’s latest Global Economic Prospects report.

Given fragile economic conditions, any new adverse development such as higher-than-expected
inflation, abrupt rises in interest rates to contain it, a resurgence of the COVID-19 pandemic, or
escalating geopolitical tensions could push the global economy into recession.

Source:
https://www.worldbank.org/en/news/press-release/2023/01/10/global-economic-prosp
ects

50. ‘The Treaty of Punakha’ significant in Indian historical context was signed between
India and-

A. Bhutan
B. Nepal
C. China
D. Sri Lanka

Correct Answer : A

Answer Justification :

Option (a) is correct: The term “friendship” is deployed as a key diplomatic category in
Bhutan’s most significant relationship with India.

However, the origin of this friendship is always traced back to the mid-twentieth-century
post-colonial period.
A much longer history of this friendship with a special focus on the landmark 1910 Treaty
of Punakha between Bhutan and Britain, which was a key turning point in Bhutan’s
relations with its southern neighbour (British India at the time).

instacourses.insightsonindia.com 47

https://upscmaterial.online/
Download From - https://upscmaterial.online/

.
Total Marks : 200.00
Test-24 (Subject)
( Insta Prelims Test Series 2023 )

The 1910 treaty was signed at a watershed moment after the then recent installation of a
monarchy in Bhutan in 1907. Throughout the twentieth century, the impact of this
friendship treaty was of paramount significance, and its shadow continues into the
twenty-first century.

Source:
https://www.cambridge.org/core/services/aop-cambridge-core/content/view/84B58A6
909F14A5B6D609B825777A0D1/S0165115322000067a.pdf/div-class-title-friendship-
and-international-relations-in-the-himalayas-bhutan-britain-and-the-1910-treaty-of-
punakha-div.pdf

51. With reference to Nord Stream Pipeline, consider the following statements:
1. It is the longest subsea pipeline which runs under the Baltic Sea exporting gas to Europe.
2. The gas for Nord Steam comes from the gas condensate deposit in Western Siberia.
3. The Nord Stream crosses the Exclusive Economic Zones and well as the territorial waters of
both Denmark and Germany.

Which of the statements given above is/are correct?


A. 1 only
B. 1 and 2 only
C. 2 and 3 only
D. 1, 2 and 3

Correct Answer : D

Answer Justification :

The Nord Stream pipeline has run into trouble from environmentalists who argue that it does
not fit in with German efforts to cut dependence on fossil fuels and fight climate change.

Statement 1 is correct: The Nord Stream pipeline owned by the Russian energy giant,
Gazprom, Nord Stream the longest subsea pipeline, is an export gas pipeline which runs under
the Baltic Sea carrying gas from Russia to Europe.

Statement 2 is correct: The gas for Nord Steam comes mainly from the Bovanenkovo oil and
gas condensate deposit in Western Siberia.

Statement 3 is correct: The Nord Stream crosses the Exclusive Economic Zones (EEZs) of
several countries including Russia, Finland, Sweden, Denmark and Germany, and the territorial
waters of Russia, Denmark, and Germany.

In Germany, the pipeline connects to the OPAL (Baltic Sea Pipeline) and NEL (North European
Pipeline) which further connects to the European grid.

Source:

instacourses.insightsonindia.com 48

https://upscmaterial.online/
Download From - https://upscmaterial.online/

.
Total Marks : 200.00
Test-24 (Subject)
( Insta Prelims Test Series 2023 )

https://www.thehindu.com/news/international/explained-the-importance-of-the-nord-
stream-pipeline/article38352754.ece

52. With reference to Teesta River Dispute, consider the following statements:
1. It is a distributary of the Barak River which flows through Assam and then flows on to
Bangladesh.
2. Bangladesh seeks an equitable distribution of Teesta waters on the lines of the Ganga Water
Treaty of 1996.
3. The Rahimpur Canal project was originally built to give access of Teesta’s water to Assam
during the lean season.

Which of the statements given above is/are correct?


A. 1 only
B. 2 only
C. 2 and 3 only
D. 1 and 3 only

Correct Answer : B

Answer Justification :

Statement 1 is not correct: Teesta river originates in the Himalayas and flows through
Sikkim and West Bengal to merge with the Brahmaputra in Assam and (Jamuna in Bangladesh),
is perhaps the most contentious issue between India and Bangladesh.

The river covers nearly the entire floodplains of Sikkim, while draining 2,800 sq km of
Bangladesh.

Kushiyara river is a distributary of the Barak river which flows through Assam, and then on to
Bangladesh.

Statement 2 is correct: Bangladesh has sought an “equitable” distribution of Teesta waters


from India, on the lines of the Ganga Water Treaty of 1996, but to no avail. The failure to ink a
deal had its fallout on the country’s politics.

The water of Kushiyara will be channelled through the Rahimpur Canal project in Sylhet.

Statement 3 is not correct: The Rahimpur Canal project in Zakiganj upazila or subdivision of
Sylhet was built to help the farmers access Kushiyara’s water but the facility used to remain dry
during the lean season without serving the purpose for which it was built.

Source:
https://www.thehindu.com/news/international/the-hindu-explains-teesta-water-shari
ng/article17894299.ece

https://www.thehindu.com/news/international/explained-why-is-the-kushiyara-river-t
instacourses.insightsonindia.com 49

https://upscmaterial.online/
Download From - https://upscmaterial.online/

.
Total Marks : 200.00
Test-24 (Subject)
( Insta Prelims Test Series 2023 )

reaty-between-india-and-bangladesh-important/article65874597.ece

53. With reference to the Eastern Economic Forum (EEF), consider the following
statements:
1. The primary objective of the EEF is to increase the Foreign Direct Investments in the region that
encompasses one-third of Russia’s territory.
2. India has abstained itself from investing in EEF due to the current international conditions
including Russia-Ukraine conflict and China’s involvement in EEF.

Which of the statements given above is/are correct?


A. 1 only
B. 2 only
C. Both 1 and 2
D. Neither 1 nor 2

Correct Answer : A

Answer Justification :

The EEF was established in 2015 to encourage foreign investments in the RFE. The EEF displays
the economic potential, suitable business conditions and investment opportunities in the region

Statement 1 correct: Primary objective of the EEF is to increase the Foreign Direct
Investments in the Russia’s Far East.

The region encompasses one-third of Russia’s territory and is rich with natural resources such
as fish, oil, natural gas, wood, diamonds and other minerals.

Although, the EEF is an annual gathering, the forum comes at an opportune time for Russia who
is dealing with the impact of the sanctions.

China is the biggest investor in the region as it sees potential in promoting the Chinese Belt and
Road Initiative and the Polar Sea Route in the RFE. China’s investments in the region account
for 90% of the total investments.

Statement 2 is not correct: During the forum, India expressed the country’s readiness
in expanding trade, connectivity and investments in Russia.
India is keen to deepen its cooperation in energy, pharmaceuticals, maritime connectivity,
healthcare, tourism, the diamond industry and the Arctic
India has vested interests in the EEF and has worked towards balancing its involvement.

Source:
https://www.thehindu.com/news/international/explained-the-eastern-economic-forum
-and-indias-balancing-act/article65894341.ece

instacourses.insightsonindia.com 50

https://upscmaterial.online/
Download From - https://upscmaterial.online/

.
Total Marks : 200.00
Test-24 (Subject)
( Insta Prelims Test Series 2023 )

54. With reference to Quadrilateral Strategic Dialogue (QUAD), consider the following
statements:
1. Quad is not a structured multilateral organisation and lacks a secretariat and any permanent
decision-making body.
2. Unlike creating policy on the lines of United Nations, Quad has focused on expanding existing
agreements between member countries.
3. Like the North Atlantic Treaty Organisation, Quad includes provisions for collective defence as
well as joint military exercises.

Which of the statements given above is/are correct?


A. 1 only
B. 1 and 2 only
C. 2 and 3 only
D. 1, 2 and 3

Correct Answer : B

Answer Justification :

Following the Indian Ocean tsunami, India, Japan, Australia, and the US created an informal
alliance to collaborate on disaster relief efforts.

In 2017, faced again with the rising threat of China, the four countries revived the Quad,
broadening its objectives and creating a mechanism that aimed to slowly establish a rules-
based international order.

Statement 1 is correct: The Quad is not structured like a typical multilateral organisation and
lacks a secretariat and any permanent decision-making body.

Statement 2 is correct: Instead of creating policy along the lines of the European Union or
United Nations, the Quad has focused on expanding existing agreements between member
countries and highlighting their shared values.

Statement 3 is not correct: Additionally, unlike NATO, the Quad does not include provisions
for collective defence, instead choosing to conduct joint military exercises as a show of unity
and diplomatic cohesion.

In 2020, the trilateral India-US-Japan Malabar naval exercises expanded to include Australia,
marking the first official grouping of the Quad since its resurgence in 2017 and the first joint
military exercises among the four countries in over a decade.

Source:
https://indianexpress.com/article/explained/quad-nations-meeting-us-china-7508605/

https://www.thehindu.com/opinion/op-ed/quad-strategic-opportunity-or-quagmire/art
icle34029799.ece

instacourses.insightsonindia.com 51

https://upscmaterial.online/
Download From - https://upscmaterial.online/

.
Total Marks : 200.00
Test-24 (Subject)
( Insta Prelims Test Series 2023 )

55. Consider the following statements regarding the International Monetary Fund:
1. It is a specialized agency of the United Nations and its membership is open to those countries
who accept its statutes.
2. It provides the Extended Fund Facility by which countries can borrow money for a short-term
balance of payments.
3. It is funded by quota subscriptions paid by member states and the size of the quota is
independent of the size of the member's economy.

Which of the statements given above is/are correct?


A. 1 only
B. 2 and 3 only
C. 1 and 2 only
D. 1, 2 and 3

Correct Answer : A

Answer Justification :

The IMF came into formal existence in 1944 following the Bretton Woods Conference.

Along with its sister organization, the World Bank, it was created to prevent economic crises
such as the Great Depression.

Statement 1 is correct: It is a specialized agency of the United Nations and is run by its 190
member countries. Membership is open to any country that conducts foreign policy and accepts
the organization's statutes.

To achieve these goals, the IMF focuses and advises on the macroeconomic policies of a
country, which impacts its exchange rate, governmental budget, money, and credit
management.

In addition, as a fund, it may offer financial assistance to nations in need of correcting balance
of payment discrepancies.

Statement 2 is not correct: The IMF is funded by quota subscriptions paid by member states.
The size of each quota is determined by the size of each member's economy.

There are three widely implemented facilities by which the IMF can lend its money.

Statement 3 is not correct: A Stand-By Arrangement (SBA) offers financing of a short-


term balance of payments, usually between 12 to 24 months, but no more than 36
months.
The Extended Fund Facility (EFF) is a medium-term arrangement by which countries can
borrow a certain amount of money, typically over four to 10 years.
The third main facility offered by the IMF is known as the Poverty Reduction and Growth
Facility (PRGF).

instacourses.insightsonindia.com 52

https://upscmaterial.online/
Download From - https://upscmaterial.online/

.
Total Marks : 200.00
Test-24 (Subject)
( Insta Prelims Test Series 2023 )

Source: https://www.investopedia.com/articles/03/030703.asp

56. Consider the following statements:


1. After the Brexit, Northern Ireland remained the only constituent of United Kingdom that shared
a land border with a European Union member.
2. Northern Ireland Protocol is a post-Brexit agreement that has created a trade border between
Northern Ireland and the rest of the United Kingdom.

Which of the statements given above is/are correct?


A. 1 only
B. 2 only
C. Both 1 and 2
D. Neither 1 nor 2

Correct Answer : C

Answer Justification :

Statement 1 is correct: After Brexit, Northern Ireland remained the UK’s only constituent that
shared a land border with an EU member, the Republic of Ireland.

EU and UK having different product standards, checks would be necessary before goods could
move from Northern Ireland to Ireland.

However, the two sides have had a long history of conflict, with a hard-fought peace secured
only in 1998 under the Belfast Agreement, also called the Good Friday agreement.

Fiddling with this border was thus considered too dangerous, and it was decided the checks
would be conducted between Northern Ireland and Great Britain. This was called the Northern
Ireland Protocol.

Statement 2 is correct: The Northern Ireland Protocol, which is a post-Brexit agreement that
created a trade border between Northern Ireland and the rest of the UK.

Under the protocol, Northern Ireland remains in the EU single market, and trade-and-customs
inspections of goods coming from Great Britain take place at its ports along the Irish Sea.

Source:
https://indianexpress.com/article/explained/explained-global/uk-supreme-court-uphol
ds-legality-of-northern-ireland-protocol-8432545/

57. Consider the following statements:


1. The boundary between China and Bhutan has never been delimited and Bhutan has no formal
diplomatic relations with China.

instacourses.insightsonindia.com 53

https://upscmaterial.online/
Download From - https://upscmaterial.online/

.
Total Marks : 200.00
Test-24 (Subject)
( Insta Prelims Test Series 2023 )

2. The Sakteng Wildlife Sanctuary in Bhutan bordering Arunachal Pradesh is the oldest disputed
territory between Bhutan and China.

Which of the statements given above is/are correct?


A. 1 only
B. 2 only
C. Both 1 and 2
D. Neither 1 nor 2

Correct Answer : A

Answer Justification :

The Sakteng wildlife sanctuary in Eastern Bhutan, Beijing has doubled down, including Bhutan’s
“Eastern sectors” to the boundary dispute between the two countries for the first time.

Statement 1 is correct: The boundary between China and Bhutan has never been delimited.
There have been disputes over the eastern, central and western sectors for a long time.

Bhutan has always maintained a discreet silence on its boundary negotiations with China, and it
does not have any formal diplomatic relations with China.

Statement 2 is not correct: According to written records, there has been no mention of
Eastern Bhutan, or Trashigang Dzongkhag (district), where Sakteng is based, that borders
Arunachal Pradesh, in 24 previous rounds of boundary negotiations held between the two
countries between 1984 and 2016.

The negotiations have not been held since the Doklam standoff between Indian and Chinese
troops in 2017.

Sakteng Wildlife Sanctuary is a territory of Bhutan and China has featured it as a disputed area.

Source:
https://www.thehindu.com/news/international/days-after-demarche-china-doubles-do
wn-on-claims-on-eastern-bhutan-boundary/article31993470.ece

58. The Davos Summit often mentioned in news is organised by-

A. World Cities Culture Forum


B. World Economic Forum
C. Amnesty International
D. Oxfam International

Correct Answer : B

instacourses.insightsonindia.com 54

https://upscmaterial.online/
Download From - https://upscmaterial.online/

.
Total Marks : 200.00
Test-24 (Subject)
( Insta Prelims Test Series 2023 )

Answer Justification :

Option (b) is correct: The World Economic Forum, mostly known for its annual summit in
Davos, Switzerland, is a non-governmental lobbying organisation founded by economist Klaus
Scwab.

The Forum strives in all its efforts to demonstrate entrepreneurship in the global public interest
while upholding the highest standards of governance.

Source:
https://indianexpress.com/article/explained/explained-global/davos-2023-interesting-
facts-about-the-event-8385623/

59. With reference to Carbon Border tax, consider the following statements:
1. It involves imposing an import duty on a product manufactured in a country with more lax
climate rules than the one buying it.
2. A group of countries including India has adopted the carbon border taxes policy at the COP27.
3. It has been criticised as it can result in market distortion and aggravate the trust deficit
amongst the participating countries.

Which of the statements given above is/are correct?


A. 1 only
B. 2 and 3 only
C. 1 and 3 only
D. 1, 2 and 3

Correct Answer : C

Answer Justification :

The Carbon Border Adjustment Mechanism is a plan from the European Union (EU) to tax
carbon-intensive products, such as iron and steel, cement, fertiliser, aluminium and electricity
generation, from 2026.

Statement 1 is correct: Put simply, the carbon border tax involves imposing an import duty
on a product manufactured in a country with more lax climate rules than the one buying it.

While its advocates, like the EU, claim the tax will benefit the environment and provide a level
playing field to companies, those opposing it call the tax unfair and protectionist.

Statement 2 is not correct: A group of countries including India has opposed the carbon
border taxes policy at the COP27 saying it could “result in market distortion”.

Statement 3 is correct: The BASIC group, comprising India, China, Brazil and South Africa,
said in a joint statement that “Unilateral measures and discriminatory practices, such as carbon
border taxes, that could result in market distortion and aggravate the trust deficit amongst
instacourses.insightsonindia.com 55

https://upscmaterial.online/
Download From - https://upscmaterial.online/

.
Total Marks : 200.00
Test-24 (Subject)
( Insta Prelims Test Series 2023 )

Parties.

Source:
https://indianexpress.com/article/explained/explained-climate/carbon-border-tax-whi
ch-india-opposed-cop27-8274506/

60. With reference to the Black Sea Grain Initiative, consider the following statements:
1. It provided for a safe maritime humanitarian corridor for Ukrainian exports.
2. It created a Joint Coordination Centre comprising representatives from the United Nations.
3. It been credited for making a substantial difference to the global cost of living crisis.

Which of the statements given above is/are correct?


A. 1 only
B. 1 and 2 only
C. 2 and 3 only
D. 1, 2 and 3

Correct Answer : D

Answer Justification :

The Black Sea Grain Initiative deal, brokered by the United Nations (UN) and Turkey, was signed
in Istanbul.

Statement 1 is correct: The deal was to provide for a safe maritime humanitarian corridor for
Ukrainian exports (particularly for food grains) from three of its key ports, namely,
Chornomorsk, Odesa and Yuzhny/Pivdennyi.

The central idea was to calm markets by ensuring an adequate supply of grains, thereby
limiting food price inflation.

Ukraine is among the largest exporters of wheat, maize, rapeseed, sunflower seeds and
sunflower oil, globally.

Its access to the deep-sea ports in the Black Sea enables it to directly approach Russia and
Europe along with grain importers from the Middle East and North Africa.

Statement 2 is correct: The deal put in place a Joint Coordination Centre (JCC), comprising
senior representatives from Russia, Turkey, Ukraine and the UN for oversight and coordination.

Statement 3 is correct: The initiative has also been credited for having made a “huge
difference” to the global cost of living crisis. All commercial ships are required to register
directly with the JCC to ensure appropriate monitoring, inspection and safe passage

Source:
https://www.thehindu.com/news/international/explained-what-is-the-black-sea-grain-

instacourses.insightsonindia.com 56

https://upscmaterial.online/
Download From - https://upscmaterial.online/

.
Total Marks : 200.00
Test-24 (Subject)
( Insta Prelims Test Series 2023 )

initiative/article66085476.ece

61. The Supply Chain Resilience Initiative often heard in news was initiated by which of
the following countries?
1. Australia
2. Japan
3. Russia
4. China
5. India

Select the correct answer using the code given below:


A. 1, 2, 3 and 4 only
B. 3 and 5 only
C. 1, 2 and 5 only
D. 2, 3 and 4 only

Correct Answer : C

Answer Justification :

Option (c) is correct: China has long practised “supply chain politics”. Japanese
entrepreneurs learnt a hard lesson when the detention of a Chinese fishing trawler captain near
the disputed Senkaku Islands resulted in the Chinese government cutting off exports of rare
earths to Japan.

In the context of greater weaponization of trade and technology India, Japan and Australia
initiated the Supply Chain Resilience Initiative (SCRI) focusing on automobiles and parts,
petroleum, steel, textiles, financial services and IT sectors.

The SCRI may be bolstered by the future involvement of France, though this might depend on
the European Union’s position. The United Kingdom has also shown interest in the SCRI.

Source:
https://www.thehindu.com/opinion/lead/resilient-supply-chains-as-a-pandemic-lesson
/article33447988.ece

62. The Gogra Post recently seen in news is located in-

A. Eastern Ladakh
B. Eastern Arunachal Pradesh
C. Chungthang Valley of Sikkim
D. Dzukou Valley of Nagaland

instacourses.insightsonindia.com 57

https://upscmaterial.online/
Download From - https://upscmaterial.online/

.
Total Marks : 200.00
Test-24 (Subject)
( Insta Prelims Test Series 2023 )

Correct Answer : A

Answer Justification :

Option (a) is correct: In a significant step towards disengagement, India and China have
ceased forward deployments at the key patrol point PP17A, also known as the Gogra Post, in
eastern Ladakh.

The disengagement agreement ensures that the Line of Actual Control (LAC) in Gogra will be
strictly observed and respected by both sides, and that there is no unilateral change in the
status quo.

Source:
https://indianexpress.com/article/india/india-china-ladakh-army-gogra-7441734/

63. The Falklands Islands disputed between the United Kingdom and Argentina are
located in-

A. North Pacific Ocean


B. South Atlantic Ocean
C. Indian Ocean
D. Southern Ocean

Correct Answer : B

Answer Justification :

Option (b) is correct: Britain and Argentina embroiled in a renewed war of words after the
residents of the disputed Falkland Islands voted overwhelmingly in favour of it staying as a
British overseas territory.

Since the 18th century, the Falkland Islands, located off the coast of Argentina in the
South Atlantic Ocean, have always been subjected to colonisation and conquests by
Britain, France, Spain and Argentina.
Prior to the 1700s, the islands were uninhabited, with France first establishing a colony
there in 1764.
The next year, when the British arrived to claim the islands for themselves, it marked the
start of a dispute that has been ongoing ever since.

Source: Falkland Islanders say yes to Britain - The Hindu

64. The Sendai Framework was recently mentioned in news in the context of-

instacourses.insightsonindia.com 58

https://upscmaterial.online/
Download From - https://upscmaterial.online/

.
Total Marks : 200.00
Test-24 (Subject)
( Insta Prelims Test Series 2023 )

A. First-Level Referral Health Facilities


B. Development of Indo-Pacific
C. Combatting global terrorism
D. Disaster Risk Reduction

Correct Answer : D

Answer Justification :

Option (d) is correct: The Sendai Framework for Disaster Risk Reduction 2015-2030 (Sendai
Framework) was the first major agreement of the post-2015 development agenda and provides
Member States with concrete actions to protect development gains from the risk of disaster.

The Sendai Framework is the successor instrument to the Hyogo Framework for Action
(HFA) 2005-2015: Building the Resilience of Nations and Communities to Disasters.
The Sendai Framework works hand in hand with the other 2030 Agenda agreements,
including The Paris Agreement on Climate Change, The Addis Ababa Action Agenda on
Financing for Development, the New Urban Agenda, and ultimately the Sustainable
Development Goals.

Source:
https://www.undrr.org/implementing-sendai-framework/what-sendai-framework

65. Consider the following countries:


1. Kenya
2. Sudan
3. Somalia
4. Tanzania

Which of the above mentioned countries comprise of the Horn of Africa?


A. 1 only
B. 3 only
C. 2 and 4 only
D. 1, 2, 3 and 4

Correct Answer : B

Answer Justification :

Option (b) is correct: The Horn of Africa region occupies Africa’s easternmost peninsula,
which extends into the Guardafui Channel, Somali Sea, and the Gulf of Aden.

The Horn of African is a region that is recognized internationally as comprising Somalia,


Ethiopia, Eritrea, and Djibouti.

instacourses.insightsonindia.com 59

https://upscmaterial.online/
Download From - https://upscmaterial.online/

.
Total Marks : 200.00
Test-24 (Subject)
( Insta Prelims Test Series 2023 )

The Horn of Africa is separated from the Arabian Peninsula by the Bab el-Mandeb Strait (the
strait connecting the Red Sea and the Gulf of Aden).

The Somali Peninsula is part of the Horn of Africa and is a term used to refer to eastern Ethiopia
and Somalia.

Source: https://www.worldatlas.com/geography/horn-of-africa.html

66. Consider the following statements regarding Indian polity


1. The death of an incumbent Prime Minister automatically dissolves the Council of Ministers.
2. The Deputy ministers are not members of the cabinet and are not given independent charge of
any department.
3. The 42nd Constitutional Amendment Act of 1976 introduced the concept of ‘Cabinet’ in the
Constitution.

Which of the statements given above is/are correct?


A. 1 and 3 only
B. 1 and 2 only
C. 2 and 3 only
D. 3 only

Correct Answer : B

Answer Justification :
As the Prime Minister is the head of the Council of Ministers, the other ministers cannot
function when the Prime Minister resigns or dies.
That is, the resignation or death of an incumbent Prime Minister automatically
dissolves the Council of Ministers and creates a vacuum.
On the other hand, resignation or death of any other Minister creates a vacancy which the
Prime Minister may or may not fill.

Hence statement 1 is correct.

The deputy ministers are third in the rank in the Council of ministers after cabinet
ministers and ministers of state
They are not given independent charge of ministries or departments
They are attached to the cabinet ministers or ministers of state and assist them
in the administrative, political and parliamentary duties.
They are not members of the cabinet and hence do not attend cabinet
meetings.

Hence statement 2 is correct

Cabinet is the highest decision making authority in Indian politico-administrative system.


It is the chief policy formulating body of the central government.

instacourses.insightsonindia.com 60

https://upscmaterial.online/
Download From - https://upscmaterial.online/

.
Total Marks : 200.00
Test-24 (Subject)
( Insta Prelims Test Series 2023 )

It is an advisory body to the president and its advice is binding on him.


Cabinet did not find a place in the original text of the Constitution.
th
It was inserted in Article 352 of the Constitution by 44 Constitutional
Amendment Act of 1978.
Article 352 defines the cabinet saying that it is the Council consisting of the Prime
Minister and other ministers of cabinet rank appointed under article 75. But it does not
describe its powers and functions.

Hence statement 3 is incorrect.

67. Consider the following statements regarding the National Commission for Scheduled
Tribes
1. It is a multi-member constitutional body whose members are elected for a period of three years.
2. The conditions of service of the members are determined by the president.
3. The Constitution has empowered the commission with powers of a Civil Court in relation to
summoning any individual from any part of India and examining him on oath.

Which of the statements given above is/are correct?


A. 1 only
B. 2 and 3 only
C. 2 only
D. 1, 2 and 3

Correct Answer : D

Answer Justification :
The National Commission for Scheduled Tribes came into existence in 2004. It is a
constitutional body directly established by Article 338A of the constitution.
It consists of a chairperson, a vice chairperson and three other members.
The term of all the members is 3 years from the date of assumption of charge.

Hence statement 1 is correct.

The members are appointed by the president by warrant under his hand and Seal.
The conditions of service and tenure of office of the members are determined by the
President

Hence statement 2 is correct.

The commission has all powers of a Civil Court while investigating any matters for
inquiring into any complaint in respect to-

Summoning and enforcing the attendance of any person from any part of India
and examining him on oath.
Receiving evidence on affidavits.

instacourses.insightsonindia.com 61

https://upscmaterial.online/
Download From - https://upscmaterial.online/

.
Total Marks : 200.00
Test-24 (Subject)
( Insta Prelims Test Series 2023 )

Requiring discovery and production of any document.


Requisitioning of any public record or copy thereof from any court or office.
Issuing summons for examination of witnesses and documents
Any other matter which the president may determine.

Hence statement 3 is correct.

68. Consider the following statements regarding Vacation Bench


1. A Vacation Bench can be appointed by the president only
2. Only the Chief Justice of India can preside over a vacation bench.
3. Any advocate could approach the Vacation Officer of the supreme court to seek relief on urgent
matters.

Which of the statements given above is/are correct?


A. 3 only
B. 2 only
C. 1 and 2 only
D. 1 and 3 only

Correct Answer : A

Answer Justification :
A Vacation Bench is a special bench of the supreme court constituted by Chief Justice of
India for hearing matters during summer vacation or Winter holidays.
The Chief Justice may appoint one or more judges to hear all matters of urgent nature
during it.
The urgent matters may include pleas concerning bail, Habeas Corpus and other
fundamental rights’ issues
Litigants can approach the Supreme Court and if the court decides that the plea is an
urgent matter, the Vacation Bench hears the case
The president has no role in constitution of a vacation bench

Hence statement 1 is incorrect.

A vacation bench is usually constituted by junior judge but a Chief Justice of India can also
head a vacation bench.
It was the 45th Chief Justice of India Deepak Mishra who constituted two vacation benches
for winter break; he also headed one of the benches himself.

Hence statement 2 is incorrect.

The supreme court in September 22 issued a circular providing that a senior


Supreme Court Registry official is specially deputed as a Vacation Officer
This officer could be approached by any advocate seeking urgent relief either
on court holidays or during after court hours

instacourses.insightsonindia.com 62

https://upscmaterial.online/
Download From - https://upscmaterial.online/

.
Total Marks : 200.00
Test-24 (Subject)
( Insta Prelims Test Series 2023 )

in such instances, a bench would be constituted, if required.

Hence statement 3 is correct.

69. Consider the following statements


1. An individual arrested under an ordinary law should be released after 24 hours unless a
magistrate authorizes his further detention.
2. It is a constitutional right of an individual arrested under an ordinary law to be defended by a
legal practitioner.

Which of the statements above is/are correct?


A. 1 only
B. 2 only
C. Both 1 and 2
D. Neither 1 nor 2

Correct Answer : C

Answer Justification :

Article 22 of the Constitution of India deals with cases of ordinary law and cases of preventive
detention law. The first part deals with rights of a person arrested or detained under an ordinary
law. This includes

right to be informed on the grounds of arrest.


right to consult and be defended by a legal practitioner.
right to be produced before a magistrate within 24 hours excluding journey
time.
Right to be released after 24 hours unless the magistrate authorises further
detention.

These rights are available only on acts of criminal or quasi criminal nature or activities
prejudicial to public interest.

According to the Supreme Court, arrest and detention included in first part of Article 22 does
not cover arrest under orders of a court, civil arrest, arrest on failure of payment of income tax
and deportation of an alien.

Hence statements 1 and 2 are correct.

70. Consider the following statements regarding National emergency


1. A resolution for national emergency declaration should be passed by both houses of the
Parliament by a special majority within one month of its proclamation.

instacourses.insightsonindia.com 63

https://upscmaterial.online/
Download From - https://upscmaterial.online/

.
Total Marks : 200.00
Test-24 (Subject)
( Insta Prelims Test Series 2023 )

2. During the time of National Emergency the normal tenure of both Lok Sabha and state
legislative assembly can be extended by a law of parliament by one year.

Which of the statements given above is/are correct?


A. 1 only
B. 2 only
C. Both 1 and 2
D. Neither 1 nor 2

Correct Answer : C

Answer Justification :
Article 352 of the Constitution deals with National Emergency.
President can declare it when security of India or a part of it is threatened by war or
external aggression or armed rebellion.
A proclamation of emergency must be approved by both Houses of the Parliament within
one month from the date of issue.
Once approved, it continues for 6 months and can be extended to an indefinite period
with parliamentary approval every six months.
A resolution approving proclamation of emergency needs to be passed by either House of
the Parliament by a special majority that is, a majority of total membership of that house
and a majority of not less than two thirds of the members of that House present and
voting.
this provision was introduced by 44th Amendment Act of 1978.

Hence statement 1 is correct

During the time of national emergency-

The life of Lok Sabha may be extended beyond its normal term of 5 years for 1 year by a
law of the Parliament.
This extension cannot continue beyond a period of six months after an emergency has
ceased to operate.
The Parliament may also extend the normal tenure of a state legislative assembly by one
year each time for any length of time, subject to a maximum period of six months after
the emergency has ceased to operate.

Hence statement 2 is correct.

71. Consider the following statements regarding removal of High Court judges
1. A removal motion for such purpose can be initiated in either House of the Parliament.
2. The charges against a judge should be inquired into by a committee headed by the Chief Justice
of India.
3. The removal motion should be passed by each house of the Parliament by a special majority.

instacourses.insightsonindia.com 64

https://upscmaterial.online/
Download From - https://upscmaterial.online/

.
Total Marks : 200.00
Test-24 (Subject)
( Insta Prelims Test Series 2023 )

Which of the statements given above is/are correct?


A. 2 only
B. 1 and 3 only
C. 2 and 3 only
D. 1 only

Correct Answer : B

Answer Justification :
A High Court judge can be removed from his office by an order of the president
the Judges Enquiry Act 1968 regulates the procedure relating to removal of a High Court
judge by the process of impeachment
Under the act, a removal motion signed by 100 members in case of Lok Sabha or 50
members in case of Rajya Sabha is to be given to the presiding officer.
If such a motion is passed by both houses then finally, the president passes an order
removing the judge

Hence statement 1 is correct.

After the speaker or chairman of respective houses admits the motion, he is to constitute
a three-member Committee to investigate into the charges.
the committee should consist of (a) chief justice or a judge of the supreme court (b) a
chief justice of a High Court and (c) a distinguished jurist.

Hence statement 2 is incorrect.

If, the committee constituted for investigation into the charges finds the judge to be guilty
of misbehaviour or suffering from incapacity, the house can take up the consideration of
the motion
After the motion is passed by each house of the Parliament by special majority,
an address is presented to the President for removal of the judge
Finally, the President passes an order removing the judge

Hence statement 3 is correct.

72. Consider the following statements regarding Financial Emergency


1. A resolution regarding financial emergency cannot be issued when the Lok Sabha has been
dissolved.
2. Once approved, the emergency continues for one financial year.
3. President can revoke financial emergency without approval of the Parliament.
4. A Direction for reduction in salaries of High Court Judges during financial emergency maybe
issued by the president only.

Which of the statements given above is/are correct?


A. 3 and 4 only

instacourses.insightsonindia.com 65

https://upscmaterial.online/
Download From - https://upscmaterial.online/

.
Total Marks : 200.00
Test-24 (Subject)
( Insta Prelims Test Series 2023 )

B. 1, 2 and 3 only
C. 1, 2 and 4 only
D. 2, 3 and 4 only

Correct Answer : A

Answer Justification :
Under Article 360, the President can proclaim financial emergency if he is satisfied that a
situation has arisen for which financial stability of the country or any part of it is
threatened.
A resolution approving proclamation of the Financial emergency under Article 360 can be
passed by either House of the Parliament by a simple majority i.e. a majority of the
members of that House present and voting.
Such a proclamation must be approved by both houses of the Parliament within 2 months
from the date of its issue
However, if the proclamation is issued at a time when the Lok Sabha has been
dissolved or the dissolution of the Lok Sabha takes place during the period of 2
months without approving the proclamation, then the proclamation survives
until 30 days from the first sitting of the Lok Sabha after its reconstitution has
already approved it.

Hence statement 1 is incorrect.

Once approved by both Houses of the Parliament, Financial emergency continues


indefinitely.
Also, repeated parliamentary approval is not required for its continuation.

Hence statement 2 is incorrect.

A proclamation of financial emergency can be revoked by the president any time by a


subsequent proclamation.
Such a proclamation for revocation does not require approval of the Parliament.

Hence statement 3 is correct.

During a proclamation of the Financial emergency the president may issue directions for
reduction in salaries and allowances of

All or any classes of persons serving the union


Judges of the supreme court and high court

Hence statement 4 is correct.

73. Consider the following statements

1. Money can be appropriated out of the Consolidated fund of India only in accordance with a

instacourses.insightsonindia.com 66

https://upscmaterial.online/
Download From - https://upscmaterial.online/

.
Total Marks : 200.00
Test-24 (Subject)
( Insta Prelims Test Series 2023 )

parliamentary law based on recommendations of Comptroller and Auditor General of India


(CAG).
2. The Contingency Fund of India is held by the finance secretary on behalf of the president.

Which of the statements given above is/are correct?


A. 1 only
B. 2 only
C. Both 1 and 2
D. Neither 1 nor 2

Correct Answer : B

Answer Justification :
The Comptroller and Auditor General of India is a Constitutional independent office
created under Article 148 of the Constitution.
He is the head of Indian Audit and Accounts Department and controls the entire financial
system of the country at both Central and state levels.
CAG has no control over the issue of money from the consolidated fund of India.
Many departments are authorised to draw money from consolidated fund by issuing
cheques without specific authority from the CAG.
CAG is concerned only at the audit stage, when the expenditure has already taken place.
No money can be appropriated (issued or drawn) out of the consolidated fund of India
without a parliamentary law.

Hence statement 1 is incorrect.

The contingency fund of India was created by the Parliament under contingency fund of
India Act 1950
It is placed at the disposal of the president and he can make advances out of it to meet
unforeseen expenditure pending its authorisation by the parliament
The fund is held by the finance secretary on behalf of the president.

Hence statement 2 is correct.

74. Consider the following statements regarding Adjournment Motion


1. Only the Lok Sabha can introduce this motion.
2. At least half of the members of the House should support this motion, which deals with a matter
of urgent public importance.
3. The discussion on such a motion should last for at least one business day.

Which of the statements given above is/are correct?


A. 1 only
B. 3 only
C. 2 and 3 only
D. 1, 2 and 3

instacourses.insightsonindia.com 67

https://upscmaterial.online/
Download From - https://upscmaterial.online/

.
Total Marks : 200.00
Test-24 (Subject)
( Insta Prelims Test Series 2023 )

Correct Answer : A

Answer Justification :
The adjournment motion is introduced in the Parliament to draw attention of the house to
a definite matter of urgent public importance
As it interrupts the normal business of the house, it is regarded as an extraordinary
device
It involves an element of censure against the government and hence the Rajya
Sabha is not permitted to make use of this device.

Hence statement 1 is correct

The motion is introduced in the Parliament to draw attention of the house to a definite
matter of urgent public importance
Such a motion needs the support of 50 members of the house to be admitted

Hence statement 2 is incorrect

The discussion on an adjournment motion should last for not less than two hours and 30
minutes.

Hence statement 3 is incorrect.

75. Consider the following statements


1. The state should secure the right to public assistance in case of unemployment and old age
2. The state should work to raise the level of nutrition and improve public health.
3. The state should provide it consumption of intoxicating drugs that are harmful for health
4. The state should promote educational and economic interests of weaker sections of the society
and protect them from social injustice.

Which of the above are included in Socialist Principles under the Directive Principles of State
Policy?
A. 1 and 2 only
B. 2, 3 and 4 only
C. 2 and 3 only
D. 1, 3 and 4 only

Correct Answer : A

Answer Justification :

The Directive Principles of State Policy can be classified under three categories –

Socialistic principles

instacourses.insightsonindia.com 68

https://upscmaterial.online/
Download From - https://upscmaterial.online/

.
Total Marks : 200.00
Test-24 (Subject)
( Insta Prelims Test Series 2023 )

Gandhian principles
Liberal Intellectual principles

The socialist principles reflects ideology of socialism. They direct the state to-

Promote Welfare of the people


Secure the people right to adequate means of livelihood, equitable distribution of material
resources, etc.
Promote equal justice and provide free Legal Aid to the poor
Secure right to work, education and public assistant in case of unemployment, old age
and sickness
Make provision for just and humane conditions for work and maternity relief
Secure a living wage a decent standard of life and social and cultural opportunities for all
workers
Take steps to secure the participation of workers in management of industries
Raise the level of nutrition and standard of living of people and to improve public health

Hence statements 1 and 2 are correct

The Gandhian principles mandates the state, among other provisions, to

Promote educational and economic interests of the SCs and STs and other weaker
sections of the society and protect them from social injustice and exploitation.
Prohibition of consumption of intoxicating drinks and drugs that are injurious to health
Organisation of village panchayat
Promotion of cottage industry
Promotion of voluntary formation, Autonomous functioning and professional management
of cooperative societies
Prohibition of slaughter of cows, calves and milch and draught cattle to improve their
breeds

Hence statements 3 and 4 are incorrect

76. Consider the following statements regarding the Central Vigilance Commission
1. The commission was established as a statutory body on recommendations of the Santhanam
Committee on Prevention on Corruption.
2. The members are appointed based on recommendation of a committee headed by the Prime
minister of India.
3. A member of this commission is not eligible for further employment under Central or state
government.
4. In case the central government does not agree with advice of the CVC on any Vigilance issue,
the reason shall be communicated to the commission.

Which of the statements given above is/are correct?


A. 2 and 4 only
B. 1, 3 and 4 only

instacourses.insightsonindia.com 69

https://upscmaterial.online/
Download From - https://upscmaterial.online/

.
Total Marks : 200.00
Test-24 (Subject)
( Insta Prelims Test Series 2023 )

C. 1, 2 and 3 only
D. All of the above

Correct Answer : D

Answer Justification :
The Central Vigilance Commission (CVC) is the apex Vigilance institution, free of control
from any executive authority.
It monitors all Vigilance activities under the central government and advice various
authorities in in Central Government organisations regarding planning, executing
performing, their vigilance work.
CVC was originally neither a constitutional nor a statutory body.
It was established in 1964 by an executive resolution of the central government based on
recommendations by the Santhanam Committee on Prevention of Corruption.

Hence statement 1 is correct.

CVC is a multi-member body consisting of a Central Vigilance commissioner and not more
than two Vigilance Commissioners.
The members are appointed by the president on recommendation of a three-member
committee.
The committee consists of the Prime minister as its head the Union minister of Home
Affairs and the leader of opposition in the Lok Sabha.
The members hold office for a term of four years or until they attain the age of 65 years.

Hence statement 2 is correct.

The three members of the CVC are not eligible for further employment under the Central
or a state government after completion of their tenure, which is 4 years or till they attain
65 years of age.

Hence statement 3 is correct.

CVC advises the central government or its authorities regarding vigilance issues.
The central government or its authorities are required to consider the advice of the CVC
and take appropriate action.
However, if the government or any of its authorities does not agree with the advice of the
commission, it shall communicate the reasons to the CVC.

Hence statement 4 is correct.

77. Consider the following statements


1. No religious instructions can be propagated by any institution maintained completely by State
funds.
2. The constitutional right to perform religious worship and exhibit one's beliefs related to religion

instacourses.insightsonindia.com 70

https://upscmaterial.online/
Download From - https://upscmaterial.online/

.
Total Marks : 200.00
Test-24 (Subject)
( Insta Prelims Test Series 2023 )

is available only to citizens of India.

Which of the statements given above is/are correct?


A. 1 only
B. 2 only
C. Both 1 and 2
D. Neither 1 nor 2

Correct Answer : A

Answer Justification :
Under Article 28, no religious instruction can be provided in any educational
institution wholly maintained out of State funds.
Article 28 of the Constitution provides freedom from attending religious instruction. Under
it-
Religious instructions cannot be provided in Institutions fully maintained out of state
funds.
But this provision does not apply to an educational institution administered by the state
but established under any endowment or trust requiring imparting of religious instruction
in the Institution.

Hence statement 1 is correct.

According to Article 25 of the Constitution, all persons are equally entitled to-

Freedom of conscience.
Right to profess one's religious beliefs and faith openly and freely
Right to practice religious worship, ceremonies, rituals, etc
Right to propagate one’s religious beliefs but it does not include conversion of a person to
one’s own religion without his will.

However, these rights are subject to public order, morality, health and other provisions relating
to Fundamental Rights.

All these rights are available to both citizens and foreigners except enemy aliens.

Hence statement 2 is incorrect.

78. Consider the following statements regarding Central Information Commission


1. No member of the Parliament or state legislature can be appointed by the president as a
member of the commission.
2. The annual report of the commission is required to be laid before each house of the Parliament.
3. The commission is empowered to direct a public authority to appoint a Public Information
Officer.

instacourses.insightsonindia.com 71

https://upscmaterial.online/
Download From - https://upscmaterial.online/

.
Total Marks : 200.00
Test-24 (Subject)
( Insta Prelims Test Series 2023 )

Which of the statements given above is/are correct?


A. 2 only
B. 1 and 3 only
C. 1 and 2 only
D. 1,2 and 3

Correct Answer : D

Answer Justification :
The Central Information Commission was established under provisions of Right to
Information act, 2005.
The commission is an independent non constitutional body which consists of a Chief
Information Commissioner and not more than 10 information commissioners. The
members are appointed by the president on recommendation of a committee consisting
of the Prime Minister (chairperson), Leader of Opposition in the Lok Sabha and a Union
Cabinet Minister nominated by the Prime Minister.
The members should not be a member of the Parliament or member of the legislature of
any state or union territory

Hence statement 1 is correct.

The commission submits an annual report to the central government regarding


implementation of the provisions of the RTI Act.
The government then places this report before each House of the Parliament.

Hence statement 2 is correct.

While conducting its inquiry, the commission has powers of a Civil Court.
It can examine any record under control of a public authority and no record can be
withheld from the commission on any grounds.
The commission has the power to direct a public authority to appoint a Public Information
Officer when none exists.

Hence statement 3 is correct.

79. Which of the following is not included in the Fundamental Duties as enshrined in the
Constitution?

A. An individual should provide opportunities for education of his child between the age of 6
and 14 years.
B. A citizen of India should protect natural environment.
C. The state should provide for instruction in mother language at the primary stage of
education to children belonging to linguistic minority group.
D. A citizen of India should safeguard public property.

instacourses.insightsonindia.com 72

https://upscmaterial.online/
Download From - https://upscmaterial.online/

.
Total Marks : 200.00
Test-24 (Subject)
( Insta Prelims Test Series 2023 )

Correct Answer : C

Answer Justification :

The Fundamental Duties are inspired by the constitution of erstwhile USSR. The 42nd
Amendment Act of 1976 added a new part, namely Part IVA to the constitution. It consists of
only one article, namely Article 51A, which specified a code of 10 Fundamental Duties of the
citizens. It provides that every citizen of India should

Provide opportunities for education to his child or ward between the age of 6 and 14
years. This duty was added by 86th Constitutional Amendment Act of 2002
Abide by the constitution
Cherish and follow Nobel ideas that inspired the national freedom movement
Uphold and protect sovereignty unity and integrity of India
Protect and improve the natural environment including forests, lakes, rivers, wildlife and
to have compassion for living creatures.
Defend the country and render national service when called upon to do so
Promote harmony and the spirit of common brotherhood among all
Strive to develop scientific temper, humanism and spirit of enquiry and reform
Value and preserve the rich heritage of the country’s composite culture
A citizen of India should safeguard public property and abjure violence.

Hence statements a, b and d are correct.

Article 350-A part XVII of the constitution provides that it shall be the endeavour of every
state and every local authority within the state to provide adequate facilities for
instruction in the mother tongue at the primary stage of education for children belonging
to linguistic minority groups
It is a directive principle and not a fundamental duty.

Hence statement c is incorrect.

80. The consequences of proclamation of National Emergency under article 352 include
1. State legislatures are suspended and they come under complete control of the Centre
2. President can issue ordinances on the state subjects.
3. President can cancel transfer of finances from the Centre to the States.

Select the correct answer using the codes given below


A. 1 and 3 only
B. 2 and 3 only
C. 2 only
D. 1, 2 and 3

Correct Answer : B

instacourses.insightsonindia.com 73

https://upscmaterial.online/
Download From - https://upscmaterial.online/

.
Total Marks : 200.00
Test-24 (Subject)
( Insta Prelims Test Series 2023 )

Answer Justification :

Proclamation of National Emergency has certain consequences on the political system of the
country. This can be grouped into three categories-

1. Effect on centre state relations


2. Effect on life of Lok Sabha and state assembly
3. Effect on Fundamental rights.
During a national emergency, the Centre can give executive directions to State on any
matter.
State governments are brought under complete control of the centre but they
are not suspended.

Hence statement 1 is incorrect

When the National emergency is in operation, Parliament can make laws on any subject
mentioned in the State list.
The legislative power of the state legislature is not suspended but it becomes subject to
overriding power of the Parliament.
President can issue ordinances on the state subjects also if Parliament is not in session.
Thus, the Constitution becomes unitary rather than federal.

Hence statement 2 is correct

During proclamation of National Emergency, the president can modify constitutional


distribution of revenues between centre and the states.
President can either reduce or cancel transfer of finances from Centre to the
states.
This continues till end of financial year in which emergency ceases to operate.

Hence statement 3 is correct.

81. With reference to the Mission SAGAR, consider the following statements:
1. It was launched by the Indian Ocean Rim Association which is aimed at strengthening the Indian
Ocean region.
2. In Mission Sagar, India became the first responder to Mauritius and Seychelles by sending life-
saving drugs.
3. It was launched to provide Covid-19 related assistance to the island nations including Maldives
and Mauritius.

Which of the statements given above is/are correct?


A. 1 only
B. 1 and 2 only
C. 2 and 3 only
D. 1, 2 and 3

instacourses.insightsonindia.com 74

https://upscmaterial.online/
Download From - https://upscmaterial.online/

.
Total Marks : 200.00
Test-24 (Subject)
( Insta Prelims Test Series 2023 )

Correct Answer : C

Answer Justification :

Statement 1 is not correct: The Indian Ocean Rim Association (IORA) is a dynamic inter-
governmental organisation aimed at strengthening regional cooperation and sustainable
development within the Indian Ocean region but Mission Sagar was launched by Government of
India.

Statement 2 is correct: In Mission Sagar, India became the first responder to Mauritius and
Seychelles by sending a consignment of life-saving drugs including hydroxychloroquine

Statement 3 is correct: Mission Sagar was launched to provide Covid-19 related assistance to
the island nations of Maldives, Mauritius, Madagascar, Comoros and Seychelles. While on the
mission, INS Kesari delivered essential food items, medicines, Ayurvedic medicines and also
deployed Medical Assistance Teams (MAT) to Mauritius and Comoros.

Source:
https://www.orfonline.org/expert-speak/mission-sagar-key-to-the-indian-ocean/

82. 1. Consider the following pairs:


Declarations in news- Adopted Objectives

1. Sirte Declaration- Peace and Security in Africa

2. Delhi Declaration Counter Terror Financing


3. Rome declaration Promote Global Tourism

How many of the pairs given above is/are correctly matched?


A. Only one pair
B. Only two pairs
C. All three pairs
D. None of the pairs

Correct Answer : B

Answer Justification :

Pair 1 is correctly matched: Sirte declaration adopted by the Thirty-fifth Ordinary Session by
Heads of State and Government in Algiers, Algeria relating to issues of peace and security on
the continent.

Pair 2 is correctly matched: The United Nations Security Council Counter-Terrorism

instacourses.insightsonindia.com 75

https://upscmaterial.online/
Download From - https://upscmaterial.online/

.
Total Marks : 200.00
Test-24 (Subject)
( Insta Prelims Test Series 2023 )

Committee’s adopted Delhi Declaration on countering the use of new and emerging
technologies for terrorist purposes.

The special meeting focused on three main areas: unmanned aerial systems, terrorist financing
and information and communications technologies.

Pair 3 is not correctly matched: During the two day G20 summit, the leaders adopted the
Rome Declaration and emphasised on Covid-19 immunisation across the globe, improving
health infrastructure, boosting economic cooperation and furthering innovation.

Source:
https://www.thehindu.com/news/national/g20-summit-fruitful-pm-modi/article372767
63.ece

https://www.un.org/securitycouncil/ctc/news/delhi-declaration-countering-use-new-a
nd-emerging-technologies-terrorist-purposes-now-available

https://archives.au.int/handle/123456789/10157

83. The Project DANTAK is sometimes talked about in context of-

A. Constructing the pioneering motorable roads in Bhutan.


B. Largest-ever infrastructure project of India in the Maldives.
C. Developing the Colombo and Trincomalee ports in Sri Lanka.
D. Building the Darchulla Bridge connecting India and Nepal.

Correct Answer : A

Answer Justification :

Option (a) is correct: The Project DANTAK strengthened the bonds of friendship between
India and Bhutan.

Project DANTAK was established on April 24, 1961 as a result of the leadership of His
Majesty the Third King and then Prime Minister Jawahar Lal Nehru.
Identifying the utmost importance of connectivity in spurring the socio-economic
development and growth of Bhutan, DANTAK was tasked to construct the pioneering
motorable roads in the Kingdom.
DANTAK completed the road connecting Samdrup Jongkhar to Trashigang in 1968. In the
same year, Thimphu was connected to Phuentsholing by DANTAK. Many Bhutanese had
also volunteered to work with DANTAK.
The medical and education facilities established by DANTAK in far flung areas were often
the first in those locations.

Source: https://pib.gov.in/PressReleaseIframePage.aspx?PRID=1714170

instacourses.insightsonindia.com 76

https://upscmaterial.online/
Download From - https://upscmaterial.online/

Total Marks : 200.00


Test-24 (Subject)
( Insta Prelims Test Series 2023 )

https://www.thehindu.com/opinion/op-ed/sovereignty-and-sensitivity/article2473190
0.ece

https://www.thehindu.com/news/international/maldives-signs-largest-ever-infrastruct
ure-project-with-indias-afcons/article36114063.ece

84. Consider the following statements:


1. Senkaku islands are located in close proximity to Taiwan’s Eastern coast.
2. Senkaku Islands are associated with geopolitical tensions in the East China Sea.

Which of the statements given above is/are correct?


A. 1 only
B. 2 only
C. Both 1 and 2
D. Neither 1 nor 2

Correct Answer : B

Answer Justification :

Statement 1 is not correct: Located 190 nautical miles away from the southwest coast of
Okinawa, the Senkaku/Diaoyu islands are in close proximity to China’s east coast and Taiwan’s
northwest coast.

Sino-Japanese relations have been severely affected by bilateral disputes, which range from
their respective interpretation of history to national positions on maritime and territorial
conflicts.

Statement 2 is correct: However, the dispute concerning sovereignty over the Senkaku
Islands has, over the past few decades, accelerated geopolitical tensions in the East China Sea
(ECS).

Presently administered by Japan, the uninhabited islands of the Senkaku chain comprise five
islets – Uotsuri Island, Kuba Island, Taisho Island, Kitakojima Island, and Minamikojima Island.

Source:
https://www.e-ir.info/2022/06/23/understanding-the-senkaku-diaoyu-islands-dispute-
diplomatic-legal-and-strategic-contexts/

85. With reference to the AUKUS Partnership, consider the following statements:
1. It is a trilateral security partnership for the Indo-Pacific region.
2. It has been initiated by Australia and includes all the QUAD members.
3. It focusses on emerging quantum technologies and undersea capabilities.

instacourses.insightsonindia.com 77

https://upscmaterial.online/
Download From - https://upscmaterial.online/

Total Marks : 200.00


Test-24 (Subject)
( Insta Prelims Test Series 2023 )

Which of the statements given above is/are correct?


A. 1 only
B. 2 only
C. 1 and 3 only
D. 2 and 3 only

Correct Answer : C

Answer Justification :

Option (c) is correct: Australia, the U.K. and the U.S. have announced a new trilateral security
partnership for the Indo-Pacific called AUKUS.

As part of this, Australia will acquire nuclear powered submarines with help from the U.K. and
the U.S.

AUKUS will also involve a new architecture of meetings and engagements between the three
countries, as well as cooperation across emerging technologies like applied AI, quantum
technologies and undersea capabilities.

Source:
https://www.thehindu.com/news/international/watch-what-is-aukus/article36585218.
ece

86. The disputed Kuril Islands sometimes talked about in news are claimed by-

A. Russia and China


B. China and Japan
C. Vietnam and Taiwan
D. Russia and Japan

Correct Answer : D

Answer Justification :

Option (d) is correct: Kuril Islands are a set of four islands situated between the Sea of
Okhotsk and the Pacific Ocean near the north of Japan's northernmost prefecture, Hokkaido.

Both Russia and Japan claim sovereignty over them though the islands have been under
Russian control since the end of World War II.
The Soviet Union had seized the islands at the end of World War II and by 1949 had
expelled its Japanese residents.
Tokyo claims that the disputed islands have been part of Japan since the early 19th
century.

instacourses.insightsonindia.com 78

https://upscmaterial.online/
Download From - https://upscmaterial.online/

Total Marks : 200.00


Test-24 (Subject)
( Insta Prelims Test Series 2023 )

Source:
https://www.thehindu.com/news/international/the-quarrel-over-kuril-islands/article65
353309.ece

87. Consider the following statements:


1. The Hague Convention is an international agreement that has been signed and ratified by India
in respect of inter-country adoptions.
2. The Montreux Convention provides international legal standards that protect the rights of all
individuals involved in migration.

Which of the statements given above is/are correct?


A. 1 only
B. 2 only
C. Both 1 and 2
D. Neither 1 nor 2

Correct Answer : A

Answer Justification :

Statement 1 is correct: The Hague Convention on Protection of Children and Co-operation is


an international convention that has been signed and ratified by India in respect of inter-country
adoptions.

The Central government has brought out regulations to ease inter-country adoptions under the
Hindu Adoptions and Maintenance Act (HAMA).

According to the Adoption (Amendment) Regulations 2021, families adopting under the Act can
receive a no-objection certificate from the government's nodal adoption body Central Adoption
Resource Authority (CARA) to take the child abroad.

Statement 2 is not correct: According to the 1936 Montreux Convention Regarding the
Regime of the Straits, often referred to simply as the Montreux Convention, Turkey has control
over both the Bosporus and Dardanelles straits.

In the event of a war, the pact gives Ankara the right to regulate the transit of naval warships
and to block the straits to warships belonging to the countries involved in the conflict.

Source:
https://frontline.thehindu.com/dispatches/explained-the-montreux-convention-and-w
hether-turkey-can-close-its-waterway-to-russian-navy/article65220209.ece

https://economictimes.indiatimes.com/news/india/government-introduces-new-regula
tions-to-make-inter-country-adoptions-easier/articleshow/86317897.cms

instacourses.insightsonindia.com 79

https://upscmaterial.online/
Download From - https://upscmaterial.online/

Total Marks : 200.00


Test-24 (Subject)
( Insta Prelims Test Series 2023 )

88. With reference to Farzad B Gas field, consider the following statements:
1. It is located between the Iranian and Saudi territories.
2. It was discovered by India’s ONGC Videsh Ltd (OVL) in 2000.
3. It has been recently handed over to India by Iran for Gas exploration.

Which of the statements given above is/are correct?


A. 1 only
B. 1 and 2 only
C. 2 and 3 only
D. 1, 2 and 3

Correct Answer : B

Answer Justification :

Statement 1 is correct: The Farzad B gas field in the Farsi region, is located between the
Iranian and Saudi territories.

Despite India’s long-standing cooperation regarding the gas field and is indicative of the impact
of the U.S. sanctions on India-Iran energy cooperation, which had been drastically reduced
during the Donald Trump administration in the U.S.

Statement 2 is correct: The ONGC Videsh Ltd (OVL) had discovered the gas field in 2000 and
has been part of the ongoing cooperation on that front.

Statement 3 is not correct: Iran is developing Farzad B gas field domestically by giving it to
Petropars, a domestic gas producer which has replaced India.

Source:
https://www.thehindu.com/news/international/iran-to-develop-farzad-b-gas-field-dom
estically-dumps-india/article34581669.ece

89. The Houthis are associated with an Islamist revivalist movement primarily based in-

A. Jordan
B. Libya
C. Yemen
D. Sudan

Correct Answer : C

Answer Justification :

instacourses.insightsonindia.com 80

https://upscmaterial.online/
Download From - https://upscmaterial.online/

Total Marks : 200.00


Test-24 (Subject)
( Insta Prelims Test Series 2023 )

Houthis are Yemen’s rebels and revivalists.

After the U.S. invasion of Iraq in 2003, when anti-Americanism was at its peak in the Muslim
world, several Islamist organisations had tried to mobilise supporters riding the public
sentiments.

For the Houthis in northern Yemen, it was a tipping point.


The Houthis, under the leadership of Hussein al-Houthi, were turning political.
When the second intifada broke out in the Palestinian territories in 2000, the Houthis
staged solidarity protests.
They mobilised supporters against the U.S.’s war on Afghanistan in 2001. After the Iraq
war, they adopted a new slogan, “Death to America, death to Israel, curse upon the Jews,
victory to Islam”.
Eventually this tiny group of tribesmen from the Marran Mountains of the northern
province Sa’dah grew into the most powerful rebel war machine in Yemen.

Source:
https://www.thehindu.com/news/international/houthis-yemens-rebels-and-revivalists/
article33644813.ece

90. Consider the following statements:


1. The Sinai Peninsula is a desert region situated between the Red Sea and the Mediterranean Sea
2. The Sinai Peninsula is bordered to the east by the Gaza Strip and the Suez Canal to the west.
3. The Egyptian army launched an operation in Sinai to get rid of aggressive branch of the Islamic
State.

Which of the statements given above is/are correct?


A. 1 and 2 only
B. 2 and 3 only
C. 1 and 2 only
D. 1, 2 and 3

Correct Answer : D

Answer Justification :

The Sinai Peninsula, which is part of Egypt, is shaped like a triangle, serves as a land bridge
between the Middle Eastern portion of Asia and North Africa.

Statement 1 is correct: This sparsely populated desert region between the Red Sea and the
Mediterranean Sea has become one of Egypt’s biggest worries and brings trouble to the
doorstep of Israel.

instacourses.insightsonindia.com 81

https://upscmaterial.online/
Download From - https://upscmaterial.online/

Total Marks : 200.00


Test-24 (Subject)
( Insta Prelims Test Series 2023 )

Statement 2 is correct: The peninsula is bordered to the north by the Mediterranean Sea, and
to the east by Israel and the Gaza Strip.

To the west of the Sinai Peninsula is the Suez Canal, across which lies the African part of Egypt.

The Sinai is bordered to the southwest by the Gulf of Suez, and to the immediate south by the
Red Sea.

The Sinai Peninsula is home to an increasingly aggressive branch of the Islamic State as well as
smaller jihadist cells.

Statement 3 is correct: The Egyptian army launched a large-scale operation dubbed “Sinai
2018” in February to rid the Sinai of jihadists after a dastardly attack on a Sufi-mosque which
killed more than 300.

Source:
https://www.thehindubusinessline.com/opinion/egypts-sinai-crisis/article25902550.ec
e

https://www.worldatlas.com/articles/where-is-the-sinai-peninsula.html

91. The Global Biofuel Alliance (GIBA) is a prioritised Initiative under-

A. National Hydrogen Energy Mission


B. India’s G20 Presidency
C. National Policy on Biofuels
D. Association of Southeast Asian Nations

Correct Answer : B

Answer Justification :

Option (b) is correct: Brazil, India, and the United States, as leading biofuel producers and
consumers, will work together towards the development of a Global Biofuels Alliance along with
other interested countries.

This Alliance will be aimed at facilitating cooperation and intensifying the use of
sustainable biofuels, including in the transportation sector.
The Alliance shall work in collaboration with and complement the relevant existing
regional and international agencies as well as initiatives in the bioenergy, bioeconomy,
and energy transition fields more broadly, including the Clean Energy Ministerial Biofuture
Platform, the Mission Innovation Bioenergy initiatives, and the Global Bioenergy
Partnership (GBEP).
The Global Biofuel Alliance is one of the priorities under India’s G20 Presidency and was

instacourses.insightsonindia.com 82

https://upscmaterial.online/
Download From - https://upscmaterial.online/

Total Marks : 200.00


Test-24 (Subject)
( Insta Prelims Test Series 2023 )

announced by Minister of Petroleum & Natural Gas and Housing & Urban Affairs during
India Energy Week 2023.

Source: https://pib.gov.in/PressReleaseIframePage.aspx?PRID=1898274

92. Consider the following statements:


1. The Indian Constitution does not allow the aldermen of a municipality the right to vote in its
meetings.
2. As per the Delhi Municipal Corporation Act of 1957, the Mayor, or in his absence the Deputy
Mayor has to preside over every meeting of the corporation.

Which of the statements given above is/are correct?


A. 1 only
B. 2 only
C. Both 1 and 2
D. Neither 1 nor 2

Correct Answer : C

Answer Justification :

“Alderman” refers to a member of a city council or municipal body, with exact responsibilities
depending on the location of its usage. It is derived from Old English.

In the 12th century CE, as guilds became increasingly associated with municipal governments,
the term came to be used for officers of municipal bodies. This is the sense in which it is used
till date.

Statement 1 is correct: The Constitution of India does not allow nominated members
(aldermen) of a municipality the right to vote in meetings

Statement 2 is correct: Since in terms of Section 76 of the Delhi Municipal Corporation Act of
1957, the Mayor, or in his absence the Deputy Mayor, has to preside over every meeting of the
corporation, the simultaneous holding elections of Mayor, Deputy Mayor and Members of the
Standing Committees is directly contrary to the provisions of the statute.

As per the Delhi Municipal Corporation Act, 1957, ten people, over the age of 25 can be
nominated to the corporation by the administrator (the Lieutenant Governor).

These people are expected to have special knowledge or experience in municipal


administration. They are meant to assist the house in taking decisions of public importance.

Source:
https://indianexpress.com/article/explained/alderman-what-does-the-term-mean-836
3873/

instacourses.insightsonindia.com 83

https://upscmaterial.online/
Download From - https://upscmaterial.online/

Total Marks : 200.00


Test-24 (Subject)
( Insta Prelims Test Series 2023 )

https://www.thehindu.com/news/cities/Delhi/supreme-court-seeks-replies-of-office-of-
delhi-lg-on-plea-for-early-holding-of-mayoral-election-in-mcd/article66484658.ece

93. Consider the following statements:


1. In India, the practice of the President addressing Parliament can be found only in the
Government of India Act of 1935.
2. The Indian Constitution gives the President the power to address only a joint sitting of the two
Houses of Parliament.
3. The speech that the President reads in the Motion of Thanks is the viewpoint of the government
and is written by it.

Which of the statements given above is/are not correct?


A. 3 only
B. 2 and 3 only
C. 1 and 2 only
D. 1, 2 and 3

Correct Answer : C

Answer Justification :

Statement 1 is not correct: In India, the practice of the President addressing Parliament can
be traced back to the Government of India Act of 1919.

This law gave the Governor-General the right of addressing the Legislative Assembly and the
Council of State.

The President’s address is one of the most solemn occasions in the Parliamentary calendar. It is
the only occasion in the year when the entire Parliament, i.e. the President, Lok Sabha, and
Rajya Sabha come together.

Statement 2 is not correct: The Constitution gives the President the power to address either
House or a joint sitting of the two Houses of Parliament. Article 87 provides two special
occasions on which the President addresses a joint sitting.

The first is to address the opening session of a new legislature after a general election. The
second is to address the first sitting of Parliament each year.

The Prime Minister replies to the motion of thanks in both Houses, and responds to the issues
raised by MPs. The motion is then put to vote and MPs can express their disagreement by
moving amendments to the motion.

Statement 3 is correct: The address of the President follows a general structure in which it
highlights the government’s accomplishments from the previous year and sets the broad
governance agenda for the coming year.

instacourses.insightsonindia.com 84

https://upscmaterial.online/
Download From - https://upscmaterial.online/

Total Marks : 200.00


Test-24 (Subject)
( Insta Prelims Test Series 2023 )

The speech that the President reads is the viewpoint of the government and is written by it.

Source:
https://indianexpress.com/article/explained/president-address-in-house-ram-nath-kov
ind-lok-sabha-rajya-sabha-7165832/

94. The species ‘Dickinsonia’ recently has been mentioned in news in context of-

A. The Earth’s oldest animal


B. The Earth’s oldest cryptogam
C. World’s oldest vertebrate found in Egypt
D. Fossil belonging to the family of Protecetids

Correct Answer : A

Answer Justification :

Option (a) is correct: Dickinsonia were among the earliest living organisms on earth.

The period in the Earth’s history when Dickinsonia and several multicellular organisms existed,
was approximately between 635 million years ago (Ma) and 541 Ma, with the living creatures of
the era called Vendobionts. Earlier, Dickinsonia fossils were found in Russia and Australia
among other places.

Dickinsonia are believed to be one of the key links between the early, simple organisms and the
explosion of life in the Cambrian Period.

Source:
https://timesofindia.indiatimes.com/city/bhopal/earliest-animal-fossil-discovered-nea
r-bhopal-turns-out-to-be-beehive/articleshow/97714550.cms

95. Consider the following statements:


1. The Atal Innovation Mission is India’s flagship initiative to promote a culture of innovation and
entrepreneurship.
2. Urban-20 is one of the Engagement Groups under the Atal Innovation Mission to facilitate urban
development in India.

Which of the statements given above is/are correct?


A. 1 only
B. 2 only
C. Both 1 and 2
D. Neither 1 nor 2

instacourses.insightsonindia.com 85

https://upscmaterial.online/
Download From - https://upscmaterial.online/

Total Marks : 200.00


Test-24 (Subject)
( Insta Prelims Test Series 2023 )

Correct Answer : A

Answer Justification :

Statement 1 is correct: Atal Innovation Mission (AIM) is Government of India’s flagship


initiative to create and promote a culture of innovation and entrepreneurship across the length
and breadth of our country.

AIM’s objective is to develop new programmes and policies for fostering innovation in different
sectors of the economy, provide platforms and collaboration opportunities for different
stakeholders, and create an umbrella structure to oversee the innovation & entrepreneurship
ecosystem of the country.

Statement 2 is not correct: Urban-20 (U20) event being organised under G20 presidency of
India.

The Event is being organised by Ministry of Housing and Urban Affairs.

Provides a platform for cities from G20 countries to facilitate discussions on various important
issues of urban development including climate change, social inclusion, sustainable mobility,
affordable housing

Source: https://pib.gov.in/PressReleaseIframePage.aspx?PRID=1885167

https://aim.gov.in/

96. The Reserve Bank of India has recently issued an Alert list to-

A. Recognise the non-performing assets of the banks.


B. Restrict the rising inflation running above its targets.
C. Create awareness against Chit Fund Business in India.
D. Caution against undertaking unauthorised forex transactions.

Correct Answer : D

Answer Justification :

Option (d) is correct: The Reserve Bank of India (RBI) has recently come out with an
updated Alert List of entities/platforms/websites which are neither authorised to deal in forex
under the Foreign Exchange Management Act, 1999 (FEMA) nor authorised to operate electronic
trading platforms (ETPs) for forex transactions.

The RBI had cautioned the members of public against unauthorised forex trading
platforms and in September 07, 2022, issued an Alert List of such entities.
On Friday the RBI in a circular said that the Alert List is not exhaustive.

instacourses.insightsonindia.com 86

https://upscmaterial.online/
Download From - https://upscmaterial.online/

Total Marks : 200.00


Test-24 (Subject)
( Insta Prelims Test Series 2023 )

An entity not appearing in the Alert List should not be assumed to be authorised by the
RBI to deal in foreign exchange or operate electronic trading platforms for forex
transactions.

Source:
https://www.thehindu.com/business/rbi-issues-updated-alert-list-to-curb-unauthorise
d-forex-trading/article66494373.ece

97. With reference to the Market Access Initiative Scheme, consider the following
statements:
1. It is strictly an Export Promotion Scheme that acts as a catalyst to promote India’s exports.
2. As a drawback, the level of assistance for each eligible activity has not been fixed under the
Scheme.
3. Only the Apex Trade Bodies recognized under Foreign Trade Policy of India are eligible agencies
under the initiative.

Which of the statements given above is/are correct?


A. 1 only
B. 1 and 2 only
C. 2 and 3 only
D. 1, 2 and 3

Correct Answer : A

Answer Justification :

Statement 1 is correct: Market Access Initiative (MAI) Scheme is an Export Promotion


Scheme envisaged to act as a catalyst to promote India’s exports on a sustained basis.

The scheme is formulated on focus product-focus country approach to evolve specific market
and specific product through market studies/survey.

Assistance would be provided to Export Promotion Organizations/Trade Promotion


Organizations/National Level Institutions/ Research Institutions/Universities/Laboratories,
Exporters etc., for enhancement of exports through accessing new markets or through
increasing the share in the existing markets.

Statement 2 is not correct: Under the Scheme the level of assistance for each eligible
activities have been fixed.

Statement 3 is not correct: Eligible Agencies

Departments of Central Government and Organisation of Central/


State Governments including
Indian Missions abroad

instacourses.insightsonindia.com 87

https://upscmaterial.online/
Download From - https://upscmaterial.online/

Total Marks : 200.00


Test-24 (Subject)
( Insta Prelims Test Series 2023 )

Export Promotion Councils


Registered trade promotion Organisation
Commodity Boards
Apex Trade Bodies recognized under Foreign Trade Policy of Govt of India

Source:
https://commerce.gov.in/international-trade/trade-promotion-programmes-and-sche
mes/trade-promotion-programme-focus-cis/market-access-initiative-mai-scheme/

98. Which one of the following India-US Defence Exercise includes combatting bio terror
attacks?

A. Exercise Malabar
B. Exercise Tarkash
C. Exercise Yudh Abhyas
D. Exercise Sampriti

Correct Answer : B

Answer Justification :

Option (b) is correct: First time, India-US exercise includes response to nuke & bio terror
attacks under the exercise Named TARKASH, the exercise by the National Security Guard (NSG)
and US Special Operations Forces (SOF).

With chemical and biological warfare being recognised as a looming threat to the world, an
ongoing Indo-US joint exercise has for the first time included “Chemical, Biological, Radiological
and Nuclear (CBRN) terror response”.

The exercise comes in the backdrop of Russian allegations against Ukraine in May last year that
Kyiv had orchestrated a chemical attack in Kharkiv to blame Russia and get military aid from
the West.

Source:
https://indianexpress.com/article/india/first-time-india-us-exercise-includes-response
-to-nuke-bio-terror-attacks-8437926/

99. With reference to the SWAYAM Programme, consider the following statements:
1. It is aimed at providing encouragement and support to especially abled children to pursue
Technical Education.
2. It is a Ministry of Human Resource Development Scheme implemented by the All India Council
for Technical Education (AICTE).
3. It conducts on-line certification courses for which exams are held every semester in a hybrid

instacourses.insightsonindia.com 88

https://upscmaterial.online/
Download From - https://upscmaterial.online/

Total Marks : 200.00


Test-24 (Subject)
( Insta Prelims Test Series 2023 )

mode.

Which of the statements given above is/are not correct?


A. 2 only
B. 3 only
C. 2 and 3 only
D. 1 and 3 only

Correct Answer : B

Answer Justification :

Statement 1 is not correct: SWAYAM is a programme initiated by Government of India and


designed to achieve the three cardinal principles of Education Policy viz., access, equity and
quality.

The objective of this effort is to take the best teaching learning resources to all, including the
most disadvantaged.

Saksham Scheme is a Ministry of Human Resource Development (MHRD) Scheme being


implemented by All India Council For Technical Education (AICTE) aimed at providing
encouragement and support to specially abled children to pursue Technical Education.

Statement 2 is not correct: The National Testing Agency (NTA) has been entrusted with the
responsibility of conducting the SWAYAM exam.

SWAYAM seeks to bridge the digital divide for students who have hitherto remained untouched
by the digital revolution and have not been able to join the mainstream of the knowledge
economy.

Statement 3 is correct: SWAYAM conducts on-line certification courses on a variety of


subjects for which exams are held every semester in the Computer Based Mode or in hybrid
mode, i.e. CBT mode and paper pen mode.

Source:
https://www.vikaspedia.in/education/policies-and-schemes/scholarships/post-matric-s
cholarship/saksham-scheme-for-economically-weaker-differently-abled-students

https://swayam.nta.ac.in/

100. The Yuva Sangam Portal has been recently launched to-

A. Build indigenous technology for deployment of renewable energy.


B. Build discipline & skill youth with military ethos in civil society.
C. Build empathy and solidarity with the Indian Soldiers and the martyrs.

instacourses.insightsonindia.com 89

https://upscmaterial.online/
Download From - https://upscmaterial.online/

Total Marks : 200.00


Test-24 (Subject)
( Insta Prelims Test Series 2023 )

D. Build empathy between youth of the North Eastern States and other States.

Correct Answer : D

Answer Justification :

Option (d) is correct: To strengthen people to people connect and build empathy between
youth of the North Eastern States and other States, Ministry of Education, Government of India
has conceptualized an initiative of ‘Yuva Sangam’ under Ek Bharat Shreshtha Bharat.

It is working in collaboration with various other ministries and departments such as


Culture, Tourism, Railways, Information & Broadcasting, Youth Affairs & Sports, Home
Affairs, Department for Development of North-East Region (DoNER) and IRCTC.
Around 1000 youth will participate in the pilot of Yuva Sangam.
The Yuva Sangam will focus on conducting exposure tours of the youth comprising of
students & off-campus youngsters from North Eastern States to other states & vice versa.
It will provide an immersive, multidimensional experience of various facets-under four
broad areas of Paryatan (Tourism), Parampara (Traditions), Pragati (Development) and
Paraspar Sampark (People-to-people connect).

Source: https://pib.gov.in/PressReleaseIframePage.aspx?PRID=1896778

instacourses.insightsonindia.com 90

https://upscmaterial.online/
Download From - https://upscmaterial.online/

www.upscmaterial.online

Join Our Official Telegram Channel

https://telegram.me/+YwhD2COlqds3N2I1

Or Scan QR Code For Join Our Channel

https://upscmaterial.online/

You might also like